Anda di halaman 1dari 81

LOCUS 1

Circles

Section - 1 EQUATIONS DESCRIBING CIRCLES

As we know very well from pure geometry, a circle is a geometrical figure described by a moving point in the
Euclidean plane such that its distance from a fixed point is always constant. The fixed point is called the centre of
the circle while the fixed distance is called the radius of the circle.
Moving over to a co-ordinate system, let us denote the centre C of the circle by (x0, y0) and the radius by r. For
any point P(x, y) lying on the circle, the length PC must be equal to r. Using the distance formula for PC, we
therefore obtain:

( x − x0 ) + ( y − y0 )
2 2
= r 2 : Equation of the circle

This equation must be satisfied by every point P(x, y) lying on the circle; therefore, this is the equation that uniquely
describes the given circle. We simply call it the equation of the circle, with centre (x0, y0) and radius r.
For example, consider the circle with its centre at (1, 1) and radius equal to unity:
y

A circle with centre


(1, 1) and r = 1

(1,1)

Fig - 01
The equation of the this circle is

( x − 1) + ( y − 1)
2 2
= 12

⇒ x2 + y2 − 2x − 2 y +1 = 0

Example – 01

Find the equation of the circle passing through the points (0, 0) , (3, 0) and (1, 2).

Mathematics / Circles
LOCUS 2

Solution: To write the equation of the required circle, we must find its centre and radius.
Recall from pure geometry that a circle can always be drawn through three non-collinear points. This
can be done as follows: join the points to form a triangle. Draw the perpendicular bisectors of any of
the two sides of this triangle. Their point of intersection gives us the centre C. The distance of C from
any of the vertices gives the radius r of the circles:
P

C is the centre of the circle passing

through P, Q, R.
C
CP = CQ = CR = r.

Q R

Fig - 02

We apply this result to the current example:


The equation of the perpendicular bisector
y of OB is
3
x=
2
A (1, 2)
The equation of the perpendicular bisector
of OA is
y −1 1
=−
1 2
C x−
2
1
x ⇒ 2y − 2 = − x
O (0,0) B (3,0) 2
Fig - 03 5
⇒ x + 2y − = 0
2
The point C is the intersection of the two angle bisectors

3 1
⇒ C ≡ , 
2 2
We can now easily evaluate the radius r as the length OC:

9 1 10
r = OC = + =
4 4 2
Finally, the equation of the required circle becomes:
2 2
 3  1  10
x−  + y−  =
 2  2 4
We will subsequently see another method to solve this type of questions.

Mathematics / Circles
LOCUS 3

Example – 02
Find the equation of the circle which touches the co-ordinate axes and whose centre lies on the line x − 2 y = 3
Solution: A circle of radius r touching the co-ordinate axes can be in one of the four following configurations,
with four corresponding equations mentioned alongside:
y y

(– r, r)
(r , r )

x x
(x – r)2 + (y – r)2 = r2 (x + r)2 + (y – r)2 = r2

y y

x x

(– r, – r)
(r – r )

2 2 2
(x – r) + (y + r) = r (x + r)2 + (y + r)2 = r2
Fig - 04
Note from these four possible cases that the centre of such a circle either lies on y = x or on y = – x.
In the current example, the centre is also given to lie on x – 2y = 3. Thus, there will be two circles,
with the two centres being given by the point of intersection y = x and y = – x with x – 2y =3.

y = x and x − 2 y = 3 ⇒ C1 ≡ ( −3,3)

Equation of the circle is ( x + 3) + ( y − 3) = 9


2 2

y = − x and x − 2 y = 3 ⇒ C2 ≡ (1, − 1)

Equation of the circle is ( x − 1) + ( y + 1) = 1


2 2

Example – 03
Find the equation of the circle with radius 5 and which touches another circle x 2 + y 2 − 2 x − 4 y − 20 = 0 externally
at the point (5, 5)
Solution: Let us first try to rearrange the equation of the given circle in the standard form from which we’ll be
able to deduce its centre and radius:

x 2 + y 2 − 2 x − 4 y − 20 = 0

( x − 1) + ( y − 2 )
2 2
⇒ = 25
Mathematics / Circles
LOCUS 4

Therefore, the centre of this circle is (1, 2) and its radius is 5.


We should now draw a geometrical figure which will certainly make things more clear:
y
( x0 , y0 )
Let the centre of the
required circle be (x0, y0).
Observe that P is the
P (5, 5) mid-pt of (1,2) and (x0, y0)
5
The required circle
(1, 2)
x

The given circle

Fig - 05

As explained in the figure, we can now evaluate (x0, y0), the centre of the required circle:

x0 + 1 y0 + 2
= 5 ⇒ x0 = 9 = 5 ⇒ y0 = 8
2 2
Thus, the required equation is

( x − 9 ) + ( y − 8)
2 2
= 25

EQUATION OF A CIRCLE : GENERAL FORM


Expanding the standard form of the equation of the circle we derived in the last section, we’ll obtain:

x 2 + y 2 − 2 x0 x − 2 y0 y + x02 + y02 − r 2 = 0

This suggests that the most general form of the equation of a circle can be written in terms of three variables; call
them g, f and c so that

2 g = −2 x0 ; 2 f = −2 y0 ; c = x02 + y02 − r 2

⇒ x0 = − g ; y0 = − f ; r 2 = x02 + y02 − c = g 2 + f 2 − c
Thus, the equation of the circle in terms of g, f and c becomes

x 2 + y 2 + 2 gx + 2 fy + c = 0 : Equation of a circle; most general form

Centre = ( − g , − f ) ; Radius = g 2 + f 2 − c
It should be apparent to you how the standard form and the general form of the circle’s equation are interconvertible.
Which form to use where is a matter of convenience and will depend on the situation.

Mathematics / Circles
LOCUS 5

As a first example, let us redo Example - 1, which involves finding the equation of the circle passing through the
points (0, 0), (3, 0) and (1, 2).

Let the equation be x 2 + y 2 + 2 gx + 2 fy + c = 0 , where g, f and c are to be determined. This equation must be
satisfied by the three points through which the circle passes, and hence we’ll obtain three equations from which
g, f and c can be determined:
Substitute (0, 0) : c=0
Substitute (3, 0) : 9 + 6g = 0
3
⇒ g=−
2
Substitute (1, 2) : 1 + 4 – 3 + 4f = 0
1
⇒ f =−
2
The required equation is hence:

x2 + y 2 − 3x − y = 0
which is the same as what we obtained in Example - 1.

Example – 04

( )
Let C be any circle with centre 0, 2 . Prove that at the most two rational points can lie on C.

Solution: By a rational point, we mean a point which has both its co-ordinates rational.

Let the equation of C be x 2 + y 2 + 2 gx + 2 fy + c = 0

We can arrive at the result easily be contradiction. Suppose that we have three rational points on the
circle with the co-ordinates (xi, yi) i = 1, 2, 3. These three points must satisfy the equation of the
circle. Thus we obtain a system of linear equations in g and f :

x12 + y12 + 2 gx1 + 2 fy1 + c = 0 ⇒  ( 2 x1 ) g + ( 2 y1 ) f + c = − ( x1 + y1 ) 


 2 2

 
x22 + y22 + 2 gx2 + 2 fy2 + c = 0 ⇒ ( 2 x2 ) g + ( 2 y2 ) f + c = ( − x22 + y22 )
x32 + y32 + 2 gx3 + 2 fy3 + c = 0 ⇒  2 x g + 2 y f + c = − x 2 + y 2 
 ( 3 ) ( 3) ( 3 3 )
The coefficients in this system of linear equations are all rational by assumption. Thus, when we solve
(
this system, we must obtain g, f and c to be all rational. But since the centre is 0, 2 , we have)
f = − 2 which gives us a contradiction.
This means that our assumption of taking three rational points on the circle is wrong ⇒ At the most
two rational points can lie on this circle.

Mathematics / Circles
LOCUS 6

Example – 05

Suppose we are given two curves C1 and C2 whose equation are as follows:

C1 : a1 x 2 + 2h1 xy + b1 y 2 + 2 g1 x + 2 f1 y + c1 = 0

C2 : a2 x 2 + 2h2 xy + b2 y 2 + 2 g 2 x + 2 f 2 y + c2 = 0
It is also given that these curves intersect in four concyclic points. Prove that
a1 − b1 a2 − b2
=
h1 h2
Solution: From the discussions in the last chapter, we know that any curve C passing through the point(s) of
intersection of two given curves C1 = 0 and C2 = 0 can be written as

C ≡ C1 + λ C2 = 0 where λ ∈ !

We can do the same in the current example to obtain the equation of the curve passing through the four
(concyclic) points of intersection as:

( a1 + λa2 ) x 2 + 2 ( h1 + λh2 ) xy + (b1 + λb2 ) y 2 + 2 ( g1 + λg 2 ) x + 2 ( f1 + λf 2 y ) + c1 + λc2 = 0


From the general form of the equation of the circle, we know that this equation (above) will represent
the equation of a circle only if:

Coeff. of x2 = Coeff. of y2 ⇒ a1 + λa2 = b1 + λb2

b1 − a1
⇒ λ=− ... (1)
b2 − a2

Coeff. of xy = 0 ⇒ h1 + λh2 = 0

h1
⇒ λ=− ... (2)
h2
From (1) and (2), we have
b1 − a1 b2 − a2
=
h1 h2

Example – 06
Suppose that the equation of a circle is

S ≡ x 2 + y 2 + 2 gx + 2 fy + c = 0
What condition must the co-ordinates of a point P(x1, y1) satisfy so that P may lie (i) inside the circle (ii) outside the
circle?

Mathematics / Circles
LOCUS 7

Solution: Let the centre of S be C and its radius be r.


The point P lies inside S if CP < r and outside S is CP > r.

From the equation of S, we know C to be (–g, – f) and r to be g 2 + f 2 − c . Using these facts, we


can easily evaluate the required conditions:

P lies inside the circle: CP 2 < r 2

( x1 + g ) + ( y1 + f )
2 2
< g2 + f 2 − c

x12 + y12 + 2 gx1 + 2 fy1 + c < 0 ... (1)

P lies outside the circle: CP 2 > r 2

( x1 + g ) + ( y1 + f )
2 2
> g2 + f 2 − c

x12 + y12 + 2 gx1 + 2 fy1 + c > 0 ... (2)

We can write (1) and (2) concisely as

P lies inside the circle ⇒ S ( x1 , y1 ) < 0


P lies on the circle ⇒ S ( x1 , y1 ) = 0
P lies outside the circle ⇒ S ( x1 , y1 ) > 0

Example – 07

Find the equation of the circle circumscribing the triangle formed by the lines x + y = 6, 2 x + y = 4 and x + 2 y = 5 .
Solution: Let us first consider the general case wherein we’ve been given three lines L1, L2 and L3 and we need
to find the circle circumscribing the triangle that these three lines form

A B L1
L2 L3

Fig - 06

Mathematics / Circles
LOCUS 8

One way to do it would be as follows:


! Find the intersection points A, B and C of the three lines
! Use these intersection points to write any two perpendicular bisectors
! Find the intersection of these two perpendicular bisectors which gives us the centre O.
! Finally, find the radius (which will equal OA, OB and OC)
This procedure will definitely become quite lengthy. We look instead for a a more elegant method.
We first try to write the equation of an arbitrary second-degree curve S passing through the intersection
points of L1, L2 and L3. Think carefully and you’ll realise that such a curve can be written in terms of
two arbitrary constants λ and µ as follows:
S ≡ L1 L2 + λ L2 L3 + µ L3 L1 = 0
That such a curve S will pass through all the three intersection points can be verified by observing that
the substitution of the co-ordinates of any of the three points in the equation above will make both
sides identically 0.
One we have such a curve, we can impose the necessary constraints to make it a circle.
Coming back to the current example, the equation of an arbitrary curve passing through the intersection
points of the three lines can be written as:
S ≡ ( x + y − 6 )( 2 x + y − 4 ) + λ ( 2 x + y − 4 )( x + 2 y − 5 ) + µ ( x + 2 y − 5 )( x + y − 6 ) = 0
To make S the equation of a circle, we simple impose the following constraints:
Coeff. of x 2 = Coeff. of y 2 ⇒ 2 + 2λ + µ = 1 + 2λ + 2µ
⇒ µ =1 .. (1)
Coeff. of xy = 0 ⇒ 3 + 5λ + 3µ = 0

⇒ λ =−6 ... (2)


5
We substitute λ and µ back into S to obtain the required equation as:
S ≡ x 2 + y 2 − 17 x − 19 y + 50 = 0
As another example of following such an approach, suppose that we are given four straight lines and
are told that they intersect at four concyclic points, as shown below:

L3
L4 D
C
A, B, C, D are four
concyclic points
B L2
A
L1
Fig - 07

Mathematics / Circles
LOCUS 9

What approach will you follow if you’re told to find the equation of the circle circumscribing this
quadrilateral? Obviously, one can always proceed by explicitly determining the centre and the radius
of the said circle, but as in the previous question, a much more elegant method exists.
Convince yourself that any second degree curve S passing through A, B, C, D can we written as

S ≡ L1 L3 + λ L2 L4 = 0

Observe carefully that the substitution of the co-ordinates of any of the four points A, B, C, D will
make both sides identically 0, implying that these four points lie on S. We now simply impose the
necessary constraint (on λ ) to make S represent a circle, thus obtaining S !

Example – 08

Consider a circle of radius r centred at the origin:

x2 + y2 = r 2
A line y = mx + c either just touches this circle or intersects it in two distinct points. What condition must m and c
satisfy?
Solution: What we need to do algebraically is solve the simultaneous system of equations

x2 + y2 = r 2 ... (1)

y = mx + c ... (2)
and find the condition on m and c for this system to have two distinct roots.
We substitute the value of y from (2) in (1):

x 2 + ( mx + c ) = r 2
2

⇒ (1 + m ) x
2 2
+ 2mc x + c 2 − r 2 = 0 ... (3)

Since the line intersects the circle (or touches it) the discriminant of (3) cannot be non-negative since
at least one real value of x must exist. Thus:

4m 2 c 2 ≥ 4 (1 + m 2 )( c 2 − r 2 )

⇒ c 2 − r 2 − m2r 2 ≤ 0

⇒ c 2 ≤ r 2 (1 + m 2 )
This is the condition that m and c must satisfy.
Incidentally, we also obtained the condition for tangency:

c 2 = r 2 (1 + m 2 )

⇒ c = ± r 1 + m2

Mathematics / Circles
LOCUS 10

Thus, y = mx ± r 1 + m2 will always be tangents to the circle x2 + y2 = r2, whatever be the value of
m. A particular case that you should observe here is that when m → ∞, the equation becomes

 y m 
lim  = x ± r
m →∞
 1+ m 1 + m2
2

⇒ x = ±r
which means that the two tangents are vertical and touch the circle at the two end-points of the
horizontal diameter. This is intuitively obvious.

Example – 09
A point P moves in the Euclidean plane in such a way that PA = λPB , where A and B are fixed points and λ > 0 .
Find the locus of P.

Solution: The easiest case is when λ = 1; then PA = PB and P will hence lie on the perpendicular bisector of AB.
We consider the case when λ ≠ 1. Let A and B be assigned the co-ordinates (a, 0) and (– a, 0) (for
convenience). This can always be done by an appropriate choice of the co-ordinate axes.
Now, let P have the co-ordinates (x, y). We have,

PA2 = λ 2 PB 2

⇒ (x − a)
2
+ y 2 = λ2 {( x + a ) + y }
2 2

⇒ (1 − λ ) x + (1 − λ ) y − 2ax (1 + λ ) x + a (1 − λ ) = 0
2 2 2 2 2 2 2

⇒ x + y − 2a
2 2 (1 + λ ) x + a = 0
2
2

(1 − λ ) 2 ... (1)

 a (1 + λ 2 ) 
This is obviously the equation of a circle centred at  1 − λ 2 , 0  . Note that this circle does not
 ( ) 
pass through either A or B.
Let us consider an example of this. Let AB be 2 units, so that we can assign (1, 0) and (–1, 0) as the
co-ordinates A and B. Let P move in such a way that PA = 2PB, i.e, λ = 2. From (1), the locus of P
is the circle:

2 (1 + 4 )
x2 + y 2 − x +1 = 0
1− 4

10
⇒ x2 + y2 + x +1 = 0
3

Mathematics / Circles
LOCUS 11

2
 5   5 4
The centre of this circle is  − , 0  and its radius is  −  + (0 ) − 1 =
2

 3   3 3
y

P
For any point P taken on the
circumference of this circle,
we will have PA = 2PB
B A
5
− ,0
(–1, 0) (1, 0)
3

The circle that we


obtained
Fig - 08

Example – 10

A fixed line L1 intersects the co-ordinate axes at P(a, 0) and Q (0, b). A variable line L2 , perpendicular to L1,
intersects the axes at R and S. Show that the locus of the points of intersection of PS and QR is a circle.
Solution: The equation of L1, using intercept form, can be written as
x y
+ =1
a b
⇒ bx + ay = ab
Since L2 is perpendicular to L1, its equation can be written as
L2 ≡ ax − by + λ = 0

where λ is a real parameter.

 λ   λ
Using the equation of L2, we can determine R and S to be  − , 0  and  0,  respectively.
 a   b
y

Q(0, b) L2
Points P and Q are fixed
whereas R and S will vary
as λ varies

λ λ
R − ,0 S 0,
a b P(a, 0)
x

Fig - 09
Mathematics / Circles
LOCUS 12

We now write the equations to PS and QR using the two-point form:


y −λ
PS : = ⇒ λx + aby = aλ .... (1)
x − a ab
y − b ab
QR : = ⇒ −abx + λy = bλ .... (2)
x λ
The relation that the intersection point of PS and QR, will satisfy can be evaluated by eliminating λ
from (1) and (2). We thus obtain
aby abx
λ= =
a− x y −b
⇒ aby 2 − ab 2 y = a 2bx − abx 2

⇒ x 2 + y 2 − ax − by = 0

a b
This represents a circle centred at  ,  and passing through the origin.
 2 2

Example – 11

 1 
Let  mi ,  , i = 1, 2, 3, 4 be four distinct points lying on a circle. Prove that m1m2 m3 m4 = 1
 mi 

Solution: We first assume an equation for this circle C, in its general form:

C : x 2 + y 2 + 2 gx + 2 fy + c = 0

 1 
Since  mi ,  satisfies the equation of C for i = 1, 2, 3, 4 we have
 mi 

1 2f
mi2 + 2
+ 2 gmi + +c = 0 i = 1, 2,3, 4
mi mi

⇒ mi4 + 2 gmi3 + cmi2 + 2 fmi + 1 = 0 i = 1, 2, 3, 4

This last equation tells us that mi' s are the roots of the following equation in m:

m 4 + 2 gm3 + cm 2 + 2 fm + 1 = 0 : Roots of this equation are


m, i = 1, 2, 3, 4

The product of the roots, which is m1m2 m3 m4 , can easily be seen to be 1 from this equation.

Mathematics / Circles
LOCUS 13

Example – 12
Find the equation of the circle C which has two fixed points A ( x1 , y1 ) and B ( x2 y2 ) as the end-points of its
diameter.
Solution: To evaluate the required equation, we can use a well known result from plane geometry: the angle in
a semicircle is a right angle.
P ( x , y)

For any point P (x, y) on


the circumference of the
A ( x 1 , y1 ) B (x 2 , y 2 ) circle, the angle APB is
a right angle

Fig - 10
Thus, we can use this fact:
(Slope of AP) × (Slope of PB) = – 1

y − y1 y − y2
⇒ × = −1
x − x1 x − x2

⇒ ( x − x1 )( x − x2 ) + ( y − y1 )( y − y2 ) = 0 ... (1)

This is the required equation that will represent C. Note that we could equivalently have used the
Pythagoras theorem in ∆APB to evaluate the equation of C:

AP 2 + PB 2 = AB 2

( x − x1 ) + ( y − y1 ) + ( x − x2 ) + ( y − y2 ) = ( x1 − x2 ) + ( y1 − y2 )
2 2 2 2 2 2

⇒ 2 x 2 + 2 y 2 − 2 x ( x1 + x2 ) − 2 y ( y1 + y2 ) = −2 x1 x2 − 2 y1 y2

⇒ ( x − x1 )( x − x2 ) + ( y − y1 )( y − y2 ) = 0
which is the same as what we obtained in (1).

Example – 13

Given the circle C : x 2 + y 2 + 2 gx + 2 fy + c = 0 , find the intercepts that it makes on the x-axis and y-axis.

Solution: We did a similar case in Example - 8 by solving simultaneously the equation of the circle and the line
on which the intercept is required. Here, we’ll proceed analogously.

Mathematics / Circles
LOCUS 14

x – intercept

C=0 To find the x-intercept,


we put y = 0 in the equation
for C :
2
x + 2gx + c = 0 ... (1)

x1 x2 This gives two values for x


x-axis (real and distinct, equal
or imaginary) x1 and x2 as shown
alongside. We need to find |x1 – x2|

x-intercept

Fig - 11

From (1), we have x1 + x2 = −2 g and x1 x2 = c.

Thus,

( x1 + x2 )
2
x1 − x2 = − 4 x1 x2

= 2 g2 − c

y - intercept

y1
We put x = 0 to get
2
y + 2fy + c = 0 ... (2)
If this has roots y1, y2, the length
y - intercept of the intercept is
2
y1 − y2 = ( y1 + y2 ) − 4 y1 y2

=2 f - c
2 ( using ( 2) )
y2

y - axis
Fig - 12

Thus, the intercepts are of length 2 g 2 − c and 2 f 2 − c respectively. Obviously, if g 2 < c the
circle and the x–axis do not touch/intersect; if f 2 < c the circle and the y–axis do not touch or
intersect.

Mathematics / Circles
LOCUS 15

Example – 14

What values can the variable a take so that the point (a – 1, a + 1) lies inside the circle x 2 + y 2 − 12 x + 12 y − 62 = 0
but outside the circle x 2 + y 2 = 8 .

Solution: We will use the general result that we derived in Example - 6 earlier (refer). Specifically, if S = 0 is the
equation of circle and P (x1 y1) be any point, then
S ( x1 , y1 ) < 0 ⇒ P lies inside S

S ( x1 , y1 ) > 0 ⇒ P lies outside S


Using these relations for the current case, we obtain

( a − 1) + ( a + 1) − 12 ( a − 1) + 12 ( a + 1) − 62 < 0
2 2
... (1)

⇒ 2a 2 − 36 < 0

⇒ −3 2 < a < 3 2 ... (i)

and ( a − 1) + ( a + 1) − 8 > 0
2 2
... (2)

⇒ 2a 2 − 6 > 0

⇒ a > 3 or a < − 3 ... (ii)


The intersection of (i) and (ii) gives us the required values of a as

(
a ∈ −3 2, − 3 ∪ ) ( 3, 3 2 )
Example – 15

Find the locus of the foot of the perpendicular drawn from the origin upon any chord of a circle
S ≡ x 2 + y 2 + 2 gx + 2 fy + c = 0 which subtends a right angle at the origin.
Solution: The situation is depicted graphically in the figure below to make things clearer:
y

B O is the origin. AB is
a particular chord of the
O x circle S which subtends a
right angle at the origin.
X(l,m ) We need to find the locus
of the foot of the
perpendicular, i.e. X(l, m).

A
Fig - 13
Mathematics / Circles
LOCUS 16

Observe that the equation of the chord AB can be written as

y − m −l
= (∵ AB ⊥ OX )
x−l m

⇒ lx + my = l 2 + m2
Now, if we homogenize the equation of S using the equation of the chord AB, what we’ll get is the
equation of the pair of straight lines OA and OB (as discussed in the last chapter on straight lines). This
is what we proceed to do:
2
 l x + my   lx + my   lx + my 
x 2 + y 2 + 2 gx  2 2 
+ 2 fy  2 2 
+ c 2 2 
= 0 ... (1)
 l +m  l +m  l +m 
This is the joint equation of OA and OB, since OA and OB need to be at right angles, we impose the
appropriate constraint for perpendicularity on (1):

Coeff. of x 2 + coeff .of y 2 = 0

   
 2 gl cl 2   2 fm cm 2 
 1 + + +
  1 + + =0
+ ( )  
2 2 + ( ) 
2 2
2 2 2 2
 l m l 2
+ m l m l 2
+ m

2 gl 2 fm c (l 2 + m 2 )
⇒ 2+ 2 + + =0
l + m2 l 2 + m2 (l 2 + m 2 )2

c
⇒ l 2 + m 2 + gl + fm + =0
2
This is the equation of a circle. To be more conventional, we should use (x, y) instead of the variables
l and m.
Thus, the required locus is

c
x 2 + y 2 + gx + fy + =0
2

Mathematics / Circles
LOCUS 17

TRY YOURSELF - I

Q. 1 Find the greatest and least distances of the point P (10, 7) from the circle x 2 + y 2 − 4 x − 2 y − 20 = 0

Q. 2 Through the origin O, a straight line is drawn to cut the line y = mx + c at P. If Q is a point on this line
such that OP ⋅ OQ = λ 2 , show that the locus of Q is a circle passing through the origin.

Q. 3 What is the area of an equilateral triangle inscribed in the circle x 2 + y 2 + 2 gx + 2 fy + c = 0?

Q. 4 Determine the equation of the circle passing through the points (1, 2) and (3, 4) and touching the line
3 x + y − 3 = 0.
Q. 5 A circle whose centre is the point of intersection of the lines 2 x − 3 y + 4 = 0 and 3 x + 4 y − 5 = 0
passes through the origin. Find its equation.

Q. 6 Find the equation of the circle which touches the x-axis and two of whose diameters lie along
2 x − y − 5 = 0 and 3 x − 2 y − 8 = 0
Q. 7 Find the equation of the circumcircle of an equilateral triangle two of whose vertices are (–1, 0) and
(1, 0) and the third vertex lies above the x–axis.
Q. 8 Find the equation of the circle passing through (1,0) and (0,1) and having the smallest possible radius.

Q. 9 The equations of the sides of a quadrilateral are given by Lr = ar x + br y + cr = 0 ( r = 1, 2,3, 4 ) .


If the quadrilateral is concyclic, show that

a1a3 − b1b3 ab +a b
= 1 3 31
a2 a4 − b2b4 a2b4 + a4b2

Q. 10 Find the point on the straight line y = 2 x + 11 which is nearest to the circle

16 ( x 2 + y 2 ) + 32 x − 8 y − 50 = 0

Mathematics / Circles
LOCUS 18

Section - 2 TANGENTS AND CHORDS

We first consider the case which we’ve already considered in Example - 8 earlier, namely, when will the line
y = mx + c be a tangent to the circle x 2 + y 2 = a 2 ?

Earlier, we solved the simultaneous system of two equations and put the discriminant of the resulting quadratic
equal to 0 to obtain the condition for tangency. Here, we follow an alternative approach.
We use the simple geometric fact that if a line L is a tangent to a circle S, then the perpendicular distance of the
centre of S from L must equal the radius of S:

S=0

L=0 If L is tangent to
S, then CP must
C equal r, the radius
of S.

Fig - 14

In the current case, the centre of x 2 + y 2 = a 2 is (0, 0) and its radius is a. The perpendicular distance of (0, 0)
from y = mx + c must equal a, i.e.
c
=a
1 + m2

⇒ c 2 = a 2 (1 + m 2 )

This is the same condition that we derived earlier. From this relation, we can infer that any line of the form
y = mx ± a 1 + m2 will always be a tangent to the circle x + y = a , whatever the value of m may be.
2 2 2

Consider now the problem of finding the equation of the tangent to the circle x 2 + y 2 = a 2 at a point P ( x1 , y1 )
lying on the circle.

P ( x 1 , y1 ) We want to find the


equation of the
tangent T at P (x1,y1)
x lying on the circle
O (0 ,0)
2 2 2
T x +y =a

Fig - 15

Mathematics / Circles
LOCUS 19

The tangent T will obviously be perpendicular to OP. Thus, the equation of T is

y − y1 − x1
=
x − x1 y1

⇒ xx1 + yy1 = x12 + y12 ...(1)

The other piece of information that we have is that ( x1 , y1 ) lies on the circle and therefore must satisfy its equation.
Thus, x12 + y12 = a 2 . Using this in (1), we obtain the equation of T as

T : xx1 + yy1 = a 2

If the point ( x1 , y1 ) has been specified in polar form, i.e. in the form ( a cos θ , a sin θ ), the equation of T becomes

T : x cos θ + y sin θ = a

We now go to the general case of finding the equation of the tangent to an arbitrary circle
S ≡ x 2 + y 2 + 2 gx + 2 fy + c = 0 at a point P( x1 y1 ) lying on this circle:
y

P ( x1 , y 1 )
We want to find the
equation of the
tangent T at P (x1, y1)
C(-g,- f ) lying on the circle S.
T

Fig - 16

Here again, as in the earlier case, we have two pieces of information which we can put to use :

P lies on S : x12 + y12 + 2 gx1 + 2 fy1 + c = 0 ...(1)

y − y1 y1 + f
T is perpendicular to CP : x − x × x + g = −1 {( x, y) is any point on T }
1 1

⇒ xx1 + yy1 + gx + fy = x12 + y12 + gx1 + fy1

We now add ( gx1 + fy1 + c) on both sides above :

Mathematics / Circles
LOCUS 20

⇒ xx1 + yy1 + g ( x + x1 ) + f ( y + y1 ) + c = x12 + y12 + 2 gx1 + 2 fy1 + c


Now we use (1) for the RHS above to finally obtain the equation of T as

T : xx1 + yy1 + g ( x + x1 ) + f ( y + y1 ) + c = 0

The expression on the left hand side of the equation above is conventionally denoted as T ( x1 , y1 )
(Note that T is a function of x and y too). Thus, the equation of the tangent can be written concisely as
T ( x1 , y1 ) = 0 : Equation of tangent at ( x1 , y1 ) to S
Using an analogous approach, we can write the equation of the normal to the circle
x 2 + y 2 + 2 gx + 2 fy + c = 0 at the point P ( x1 , y1 ) . You are urged to do this yourself. Note that
every normal of a circle will pass through the circle’s centre.

Example – 16
Find the equation of the tangent to
 1 2
(a) x + y = 1  3 3 
2 2
at  ,
 
1 3
(b) x 2 + y 2 − 2 x − 4 y + 4 = 0 at  2 , 2 + 2 
 
Solution (a) The equation of the tangent to x 2 + y 2 = a 2 at ( x1 , y1 ) is, as obtained in the preceeding discussion,
T ( x1 , y1 ) = 0
⇒ xx1 + yy1 = a 2

 1 2
Here, ( x1 , y1 ) =  ,  and a = 1 . Thus, the required equation is x + 2 y = 3
 3 3

(b) The equation of the tangent to x 2 + y 2 + 2 gx + 2 fy + c = 0 at ( x1 , y1 ) is


T ( x1 , y1 ) = 0
⇒ xx1 + yy1 + g ( x + x1 ) + f ( y + y1 ) + c = 0
1 3
Here, ( x1 , y1 ) is  2 , 2 + 2  and thus, the required equation is
 
x  3  1  3
+  2 +  y − 1 x +  − 2  y + 2 + +4 =0
2  2   2  2 

−x 3y  1
⇒ + − 3 +  = 0
2 2  2

⇒ − x + 3 y − (2 3 + 1) = 0
Mathematics / Circles
LOCUS 21

Example – 17

What is the length of the tangent to S ≡ x 2 + y 2 + 2 gx + 2 fy + c = 0 drawn from an external point P( x1 , y1 ) ?

Solution :
A
O (-g, -f ) is the centre of S.
P (x1, y1) is an external point.
Note that two tangents, PA and
P(x1,y1) PB can drawn from P to S.
O(- g,-f )
Also, PA = PB

Fig - 17

The length PA (or PB) can be evaluated by a simple application of the Pythagoras theorem.

In ∆PAO, observe that

PA2 = PO 2 − AO 2

= {( x1 + g )2 + ( y1 + f )2 } − { }
2
g2 + f 2 − c {∵ AO is the radius}

= x12 + y12 + 2 gx1 + 2 fy1 + c ...(1)

The equation of the circle being represented by S = 0, we can denote the RHS obtained in (1) by
S ( x1 , y1 ). Thus, the length of the tangent can be written concisely as

PA = S ( x1 , y1 )

For example, the length of the tangent from (4, 4) to x 2 + y 2 − 2 x − 4 y + 4 = 0 will be

l = 42 + 42 − 2 × 4 − 4 × 4 + 4

= 12

=2 3

In the next example, we discuss how to write the equation to the pair of tangents PA and PB.

Example – 18

From an external point P( x1 , y1 ), (two) tangents are drawn to the circle S ≡ x 2 + y 2 + 2 gx + 2 fy + c = 0 . These
tangents touch the circle at A and B. Find the joint equation of PA and PB.

Mathematics / Circles
LOCUS 22

Solution : Consider any point ( h, k ) lying on the tangents drawn from P to S.

(h 1 k )
A Assume (h, k) to lie
on one of the two
tangents PA or PB
P(x1,y1) (-g,-f )

Fig - 18

Since we know two points on the line PA, we can use the two-point form to write its equation :

y − k y1 − k
=
x − h x1 − h

⇒ x( y1 − k ) − y( x1 − h) = h( y1 − k ) − k ( x1 − h)

⇒ x( y1 − k ) − y( x1 − h) + (kx1 − hy1 ) = 0

Since PA is a tangent to S, its distance from the centre of the circle, ( − g , − f ), must equal the radius.
This gives

(− g ( y1 − k ) + f ( x1 − h) + kx1 − hy1 ) 2
= g2 + f 2 − c ...(1)
( x1 − h) + ( y1 − k )
2 2

To write the equation of the pair of lines in conventional form, we use ( x, y ) instead of ( h, k ) in (1),
above. Subsequent (lengthy !) rearrangements give:

{xx1 + yy1 + g ( x + x1 ) + f ( y + y1 ) + c} = ( x 2 + y 2 + 2 gx + 2 fy + c )( x12 + y12 + 2 gx1 + 2 fy1 + c )


2

The left hand side can be written concisely as (T ( x1 , y1 )) 2 as described earlier whereas the right hand
side can be written concisely as S ( x, y ) S ( x1 , y1 ). Thus, the equation to the pair of tangents can be
written concisely as

T 2 ( x1 , y1 ) = S ( x, y ) S ( x1 , y1 )

This relation be written in an even shorter form as simply T 2 = SS1 .

Mathematics / Circles
LOCUS 23

Example – 19

Find the equation to the pair of tangents drawn from the origin to the circle x 2 + y 2 − 4 x − 4 y + 7 = 0 .

Solution: We use the relation obtained in the last example, T 2 = SS1 , to write the desired equation. Here,
( x1 , y1 ) is (0, 0) while g = −2, f = −2 and c = 7 . Thus the joint equation is

T 2 (0, 0) = S ( x, y ) S (0, 0)

⇒ (−2 x − 2 y + 7)2 = ( x 2 + y 2 − 4 x − 4 y + 7)(7)

⇒ 4 x 2 + 4 y 2 + 49 + 8 xy − 28 x − 28 y = 7 x 2 + 7 y 2 − 28 x − 28 y + 49

⇒ 3 x 2 − 8 xy + 3 y 2 = 0
As expected, since the tangents have been drawn from the origin, the obtained equation is a homogenous
one.

Example – 20

Refer to Fig - 18. Suppose that A and B, the points of contact of the two tangents, are joined. What will be the
equation to this chord of contact ?

Solution: Assume the co-ordinates of A and B to be ( xA , y A ) and ( xB , yB ) respectively. The equations of the
tangents at A and B are, respectively,

xxA + yy A + g ( x + x A ) + f ( y + y A ) + c = 0
xxB + yyB + g ( x + xB ) + f ( y + yB ) + c = 0 } ...(I)

Since the two tangents intersect at P, the co-ordinates of P must satisfy the system of equations I.
Thus,

x1 x A + y1 y A + g ( x1 + x A ) + f ( y1 + y A ) + c = 0
x1 xB + y1 yB + g ( x1 + xB ) + f ( y1 + yB ) + c = 0 } ...(II)

Now, if you observe the system II carefully, you will realise that we can think of it this way : ( xA , y A )
and ( xB , yB ) are two points which satisfy the linear equation

xx1 + yy1 + g ( x + x1 ) + f ( y + y1 ) + c = 0 ...(1)

This is because substitution of ( xA , y A ) or ( xB , yB ) into (1) results in the system of equations II.
Thus (1) must be the equation of the chord of contact we are looking for, since both A and B
satisfy (1).

Mathematics / Circles
LOCUS 24

We can write this obtained equation concisely as

T ( x1 , y1 ) = 0 : Equation of the chord of contact

As an example, the chord of contact for the two tangents drawn from the origin to the circle
x 2 + y 2 − 4 x − 4 y + 7 = 0 will be

xx1 + yy1 + g ( x + x1 ) + f ( y + y1 ) + c = 0 where ( x1 , y1 ) = (0, 0)

⇒ −2 x − 2 y + 7 = 0

7
⇒ x+ y =
2

Example – 21

From an external point P, a line is drawn intersecting a circle S in two distinct points A and B. A tangent is also
drawn from P touching the circle S at T. Prove that PA ⋅ PB is always constant, and equal to PT 2 .

Solution: Let the equation of the circle be S ≡ x 2 + y 2 + 2 gx + 2 fy + c = 0 and the point P be ( x1 , y1 ) :

T We need to prove
that PA • PB = PT 2
P(x1,y1) B
A

S
Fig - 19

Notice that the line PAB passes through the fixed point P. What is variable about it is its slope, which
we assume to be tan θ . Thus, using the polar form for the lines, we obtain the equation of PAB as

x − x1 y − y1
= =r ...(1)
cos θ sin θ

In particular, the co-ordinates of A and B can be obtained in terms of θ using the value of r as PA and
PB respectively in (1).

Using (1), we can write any point on the line PAB as ( x1 + r cos θ , y1 + r sin θ ). If this point lies on the
circle, it must satisfy the circle’s equation :

( x1 + r cos θ )2 + ( y1 + r sin θ )2 + 2 g ( x1 + r cos θ ) + 2 f ( y1 + r sin θ ) + c = 0

Mathematics / Circles
LOCUS 25

⇒ r 2 + (2 g cos θ + 2 f sin θ ) r + x12 + y12 + 2 gx1 + 2 fy1 + c = 0 ...(2)

The equation (2) in r will have two roots r1 and r2 corresponding to PA and PB since A and B lie on
the circle.
Thus,

PA ⋅ PB = r1 ⋅ r2 = x12 + y12 + 2 gx1 + 2 fy1 + c

= S ( x1 , y1 )

= PT 2 (using the result obtained in Example - 17)


You are urged to prove this result using ‘pure’ geometry.

Example – 22

From an external fixed point P (h, k ), tangents are drawn to the circle x 2 + y 2 = a 2 . Find the area of the triangle
formed by these tangents and their chord of contact.
Solution:

AB is the chord of contact


A for the tangents drawn from
2 2 2
P to x + y = a
Observe that the area of
C O ∆ PAB can be written
(0,0)
P(h,k) as 1 •AB • PC
2
B

Fig - 20
Observe that ∆OAC is similar to ∆OAP . Thus, the ratio of the corresponding sides is equal.

OC AC OA
= =
OA AP OP

OA is simply the radius a, while OP is h 2 + k 2 . Thus,

OA2 a2
OC = =
OP h2 + k 2

Mathematics / Circles
LOCUS 26

Now, AC can be evaluated using Pythagoras theorem in ∆OAC :

AC = OA2 − OC 2

a4
= a2 −
h2 + k 2

h2 + k 2 − a2
=a
h2 + k 2

The area of ∆PAB is

1
∆ = × AB × PC
2

1
= × (2 AC ) × (OP − OC )
2

a (h 2 + k 2 − a 2 )3/ 2
=
h2 + k 2
____________________________________________________________________________________

Observe that this question was solved mostly through ‘pure’ geometrical considerations. A pure co-ordinate
approach for the area of the triangle can be followed but you can expect it to be much longer. Thus, in this subject,
you have to put your intuitive skills to the best use possible to determine the shortest approach.
Let us consider another example here itself which will show why a pure geometrical approach is better sometimes
than using co-ordinates.
Consider two circles with the following equations :

S1 ≡ x 2 + y 2 + 2 gx + 2 fy + c = 0

S 2 ≡ x 2 + y 2 + 2 gx + 2 fy + c sin 2 α + ( g 2 + f 2 ) cos 2 α = 0

These two circles are obviously concentric, centred at (− g , − f ). The radius of S1 is r1 = g 2 + f 2 − c while

that of S2 is r2 = g 2 + f 2 − c sin α = r1 sin α so that r1 > r2 .

The problem is as follows : from any point on S1 , two tangents are drawn to S2 . What is the angle between these
two tangents.

Mathematics / Circles
LOCUS 27

Obviously, this angle should be the same for any point chosen on S1 since the circles are perfectly
symmetrical figures. Let us draw a diagram corresponding to this situation :

P Note that
O PO = r1
OT = r2 = r1 sin α
S
Thus, ∠ OPT = α
⇒ TPS = 2 ∠ OPT = 2 α

Fig - 21

As explained in the diagram, the angle between the two tangents can be evaluated by simple geometric
considerations to be 2α . Although this example is more or less trivial, the contrast between pure
geometric and co-ordinate approach will become more apparent in some subsequent examples.

Example – 23 DIRECTOR CIRCLE


A point P moves in such a way so that the tangents drawn from it to the circle x 2 + y 2 = a 2 are perpendicular.
Find the locus of P.
Solution: To contrast between the various alternatives available to us, we will use all of them here:
A PURE-GEOMETRIC APPROACH:

T
a
P
O
(0,0)
S

Fig - 22
From the figure, it is apparent that OTPS is a square since all the angles are right angles and
OS = OT = a .
Thus, OP = 2 a , i.e., the distance of P from O is always 2 a, i.e., P lies on a circle of radius.
2 a . Thus, P satisfies the equation
x 2 + y 2 = 2a 2
This circle is called the Director circle of the given circle.

A CO-ORDINATE APPROACH - I
Let P be the point ( h, k ). The equation of the pair of tangents drawn from P to the circle is
T 2 = SS1
⇒ (hx + ky − a 2 ) 2 = ( x 2 + y 2 − a 2 )(h2 + k 2 − a 2 )
Mathematics / Circles
LOCUS 28

This combined equation will represent a pair of perpendicular straight lines if

Coeff. of x 2 + Coeff. of y 2 = 0

⇒ (k 2 − a 2 ) + (h2 − a 2 ) = 0

⇒ h 2 + k 2 = 2a 2

Using ( x, y ) instead of ( h, k ), we obtain the locus of P in conventional form :

x 2 + y 2 = 2a 2

A CO-ORDINATE APPROACH - II
The equation of any tangent to the circle x 2 + y 2 = a 2 can be written as y = mx + a 1 + m 2 .

If this line passes through P (h, k ), the co-ordinates of P must satisfy this equation:

k = mh + a 1 + m 2

⇒ (k − mh) 2 = a 2 (1 + m 2 )

⇒ (h 2 − a 2 )m 2 − 2mhk + (k 2 − a 2 ) = 0

This is a quadratic in m and will yield two values, say m1 and m2 , which physically corresponds to
the fact that two tangents can be drawn from P (h, k ) to the circle.

The two tangents are at a right angle if m1m2 = −1

k 2 − a2
⇒ = −1
h2 − a 2

⇒ h 2 + k 2 = 2a 2

Using ( x, y ) instead of k (h, k ) we obtain the locus of P:

x 2 + y 2 = 2a 2
If this question were to be encountered in an exam, the pure-geometric approach would certainly turn
out to be the fastest !

Example – 24

C1 and C2 are two concentric circles, the radius of C2 being twice that of C1 . From a point P on C2 , tangents
PA and PB are drawn to C1. Prove that the centroid of ∆PAB is on C1.

Mathematics / Circles
LOCUS 29

Solution: Let us again attempt this question using both a pure-geometric and a co-ordinate approach.
PURE-GEOMETRIC APPROACH:
Recall the following straightforward theorem pertaining to right-angle triangles.
A

B C
Fig - 23
∆ ABC is right-angled at B and BD is the median drawn from B to the opposite side AC. Then our
theorem tells us that
BD = AD = CD
This can be proved using simple geometry
We will put this theorem to use in the current example.
P

D
A
E B

C1
C2

Fig - 24
Since the radius r2 of C2 is twice that of C1 (r1) we have
PO = 2 OD
⇒ OD = PD. ... (1)
Thus,D is the mid-point of PO. This means that in ∆ OAP, AD is the median to OP. By the theorem
mentioned above, we have
AD = OD = PD = r1

Thus, in quadrilateral ADBO, we have AD = BD = OA = OB = r1. In other words, ADBO is a


1
parallelogram so E is the midpoint of OD. Thus, ED = r1 / 2 = OD ...(2)
2
Also, since AE = EB , PE is the median of ∆PAB. Thus the centroid lies on PE.
From (1) and (2), we finally obtain PD = 2 ED. Thus, D divides the median PE in the ratio
2 : 1 implying D is the centroid which lies on C1.

Mathematics / Circles
LOCUS 30

The descriptive nature of the pure-geometric solution just provided might make it appear to be very
long but actually only a few simple elementary geometry facts have been used.

CO-ORDINATE APPROACH
There’s no loss of generality in assuming that the two circles are centred at the origin. Thus, we can
write their equations as
C1 : x 2 + y 2 = r 2
C2 : x 2 + y 2 = 4 r 2

Assume the point P to have the co-ordinates ( h, k ). The equation of AB (the chord of contact) can
then be written as
Equation of AB : hx + ky = a 2
We can evaluate the co-ordinates of A( x1 , y1 ) and B( x2 , y2 ) by simultaneously solving the equations
for C1 and AB.

Thus, x1 and x2 will be the roots of


2
 r 2 − hx 
x +
2
 =r
2

 k 

⇒ (h 2 + k 2 ) x 2 − 2r 2 hx + r 2 (r 2 − k 2 ) = 0
2r 2 h r 2 (r 2 − k 2 )
⇒ x1 + x2 = and 1 2
x x = ... (3)
h2 + k 2 h2 + k 2
y1 and y2 will be the roots of
2
 r 2 − ky 
 +y =r
2 2

 h 

⇒ (h 2 + k 2 ) y 2 − 2r 2 ky + r 2 (r 2 − h 2 ) = 0
2r 2 k r 2 (r 2 − h 2 )
⇒ y1 + y2 = and 1 2
y y = ... (4)
h2 + k 2 h2 + k 2
If we let (t , s ) be the co-ordinates of the centroid G of ∆PAB, we have

x1 + x2 + h  2r 2  
t= ⇒ 3t = h  1 + 2 2 
3  h + k  
 Using (3) and ( 4 )
y1 + y2 + k  2r  
2
s= ⇒ 3s = k  1 + 2 2 
3  h + k  
Finally, observe that
2
 2r 2 
9(t + s ) = ( h + k )  1 + 2
2 2 2 2
2 
 h +k 
But since ( h, k ) lies on C2 , we have h 2 + k 2 = 4r 2 .

Mathematics / Circles
LOCUS 31

Thus,

9
9(t 2 + s 2 ) = (4r 2 )   = 9r 2
4
⇒ t 2 + s2 = r 2
implying that G (t , s ) lies on C1.

Since now you are in a good position to compare the two approaches, which one could you have
rather chosen for this question, the pure-geometric one or the co-ordinate one!

Example – 25

Consider the circle x 2 + y 2 = a 2 . A chord of this circle is bisected at the point P( x1 , y1 ). What is the equation of
this chord ?

Solution : Convince yourself that such a chord will be unique, since it must be perpendicular to the line joining the
origin to P( x1 , y1 ), as is clear from the figure below:

The chord AB is
O (0,0) bisected at P(x1,y1).
Thus, we must have
OP ⊥ AB.
B
A P(x1 y1)
Fig - 25

− x1
The slope of AB then becomes so its equation can be written simply as
y1
x1
y − y1 = − ( x − x1 )
y1
⇒ xx1 + yy1 = x12 + y12
To make this equation look “better”, we subtract a 2 from both sides

xx1 + yy1 − a 2 = x1 2 + y1 2 − a 2
so that it can now be written concisely as

T ( x1 , y1 ) = S ( x1 , y1 )
Of course, this is easily generalised to the case when the equation of the circle is in the general form
x 2 + y 2 + 2 gx + 2 fy + c = 0; the result obtained is the same. The next example discusses a good
application of this concept.

Mathematics / Circles
LOCUS 32

Example – 26

Find the locus of the mid-point of the chords of the circle x 2 + y 2 = a 2 which subtend a right angle at the centre.
Solution:
y

A M(h AB is a chord of the circle


,k) which subtends a right
B
angle at the centre.
M (h, k) is the mid-point
x of AB. We need to find the
O (0,0)
locus of M.

Fig - 26
Since AB is bisected at M ( h, k ), we can use the result obtained in the last example to write the
equation of AB:
T ( h, k ) = S ( h, k )
⇒ hx + ky − a 2 = h 2 + k 2 − a 2
⇒ hx + ky = h 2 + k 2
We can view the chord AB as a line intersecting the curve (circle) x 2 + y 2 = a 2 . Thus, we can obtain
the joint equation of OA and OB by homogenizing the equation of the circle using the equation of the
chord AB:
2
 hx + ky 
x + y −a  2
2 2 2
2 
=0
h +k 
Joint equation of
OA and OB:
⇒ (h 2 + k 2 ) 2 ( x 2 + y 2 ) − a 2 (hx + ky ) 2 = 0 ...(1)
Since OA and OB are perpendicular, we must have
Coeff .of x 2 + Coeff. of y 2 = 0 in (1)
⇒ 2(h 2 + k 2 )2 − a 2 h 2 − a 2 k 2 = 0
a2
⇒ h2 + k 2 =
2
Using ( x, y ) instead of ( h, k ), we obtain the following equation as the locus of M:

a2
x +y =
2 2

2
This is a circle concentric with the original circle as might have been expected.
Try solving this question using pure geometric considerations; the solution will be much simpler.

Mathematics / Circles
LOCUS 33

Example – 27

Consider two circles with the following equations:


C1 : x 2 + y 2 − 2 x − 2 y + 1 = 0
C2 : x 2 + y 2 − 16 x − 2 y + 61 = 0
Find the values that a can take so that the variable line y = 2 x + a lies between these two circles without touching
or intersecting either of them.

Solution : Observe carefully that what is variable about the variable line y = 2 x + a is not its slope but its
y-intercept a. Thus, we can always adjust a so that line stays between the two circles. The following
diagram makes this clear.

y The lines can vary in


this range so that they stay
between the two circles.
Thus as long on
amin < a < a max
C1
the line y = 2x + a stays
(1,1) between the two circles
(8,1)
x
C2
(0, amax)

(0, amin)
Fig - 27

Evaluate amax : The line y = 2 x + amax is a tangent to C1 if the perpendicular distance of the
centre (1, 1) of C1 from this line is equal to C1' s radius which is 1. Thus :

2 − 1 + amax
=1
5
⇒ 1 + amax = ± 5

since from the figure we 


 
⇒ amax = − 5 − 1  can see that amax is definitely 
negative 

Mathematics / Circles
LOCUS 34

Evaluate amin : The distance of C2' s center (8, 1) from y = 2 x + amin must be equal to its radius
which is equal to 2. Thus :
16 − 1 + amin
=2
5
⇒ 15 + amin = ±2 5
 we have selected the larger of the two 
 values possible since that is what 
⇒ amin = 2 5 − 15 corresponds to a , i.e. because the 
 min 
 line y = 2 x + amin lies above C2 . 
Thus, we obtain the allowed values of a as
2 5 − 15 < a < − 5 − 1

Example – 28

A circle touches the line y = x at a point P such that OP = 4 2 where O is the origin. The circle contains the
point (–10, 2) in its interior and the length of its chord on the line x + y = 0 is 6 2. Determine the equation of the
circle.
Solution: As always, before starting with the solution, it is a good to draw a diagram of the situation described
to get a feel of it. Also, as far as possible, we should try to use pure-geometric considerations to cut
down on the (complicated) algebraic manipulations that would result otherwise.
y
y=x
It should be more or
6 less apparent that to be able
2
B to contain the point (-10, 2)
inside it, the centre of
the circle must lie somewhere
A in the shaded region.
X O x The point P is then (- 4, - 4)
since OP = 4 2
Assume the centre to be at
X(h, k). The radius of this circle is
P(-4,-4) then given by r = XP where
r2 = (h + 4)2 + (k + 4)2

x+y=0

Fig - 28
' We have,
PX ⊥ ( y = x )
k+4
⇒ = −1 ...(1)
h+4
⇒ h + k +8 = 0 ...(2)

Mathematics / Circles
LOCUS 35

Also,

{ Perpendicular dist.
of X from x + y = 0 }
= BX = AX 2 − AB 2 = r 2 − AB 2

h+k
⇒ = (h + 4) 2 + (k + 4) 2 − (3 2) 2 ...(3)
2
Using (1) and (2) in (3), we obtain
(4 2) 2 = 2(h + 4) 2 − 18
⇒ h + 4 = ±5
⇒ h = −9,1
Given the region in which X lies, h must be –9. Thus, from (2), k is 1 and the radius r is 5 2 .
The required equation is therefore
( x + 9) 2 + ( y − 1) 2 = (5 2) 2
⇒ x 2 + y 2 + 18 x − 2 y + 32 = 0 ...(4)
For students used to rigor, it can finally be verified that the circle given by (4) does indeed contain the
point (-10, 2) and thus our initial assumption of the region in which the centre X lies, was correct.

Example – 29

Let C1 and C2 be two circles with C2 lying inside C1. A circle C lying inside C1 touches C1 internally and C2
externally. Determine the locus of the centre of C.

Solution: First of all, you must note that C2 is not concentric with C1. All that is said is that C2 lies somewhere
inside C1.

Let us first discuss what all would be involved in solving this question through a co-ordinate approach.
We could first assume a co-ordinate axes, say, with the x-axis lying along the line joining the centres of
C1 and C2 and the origin at the centre of C1 :

The centre of C2 can be assumed


C1 to be (a, 0) as shown.
The equations of C1 and C2
can then be written as
x 2 2
C1 : x + y = r12
O(0,0) (a, 0)
2 2
C2 C2 : (x – a) + y = r22

Fig - 29

Mathematics / Circles
LOCUS 36

We could then assume the equation of C to be x 2 + y 2 + 2 gx + 2 fy + c = 0 and impose the necessary


constraints on g, f and c so that C touches C2 externally and C1 internally. Recall the appropriate
constraints for two circles S1 and S2 touching externally and internally.

O1 O2
O1 r1 r2 r1
O2

r2

O1 O2 = r1 + r2 O1O2 =| r1 – r2 |
(Circles touch externally) (Circles touch internally)
Fig - 30
Imposing these constraints gives us the necessary conditions that g, f and c must satisfy and hence the
locus of the centre of C which is (− g , − f ).
However, we will be much letter off in using a pure-geometry approach here also as you’ll soon see.
Assume an arbitrary circle C of radius r inside C1 with centre X, and which satisfies the given constraint:
A

C X
B The circle C touches C1 internally at
A and C2 externally at B. X is the centre
O1 O2 of C while, O1 and O2 are centres of
C1 and C2 respectively. The radii of
C2
C, C1 and C2 are r, r1 and r2
respectively.

C1
Fig - 31
By the properties of circles touching internally and externally, we have
O1 X = O1 A − AX = r1 − r
O2 X = O2 B + BX = r2 + r
⇒ O1 X + O2 X = r1 + r2 ...(1)
(1) simply states the centre of C i.e. X, moves in such a way so that the sum of its distances from O1
and O2 is constant. Thus, X must lie on an ellipse with O1 and O2 as the two foci !
To sketch the path that X can take, we can follow the approach described in the unit on Complex
numbers. Fix two pegs at O1 and O2 and tie a string of length r1 + r2 between these two pegs. Use
your pencil and the taut string as a guide to trace out the ellipse. This is the path on which the centre of
C can move.
Mathematics / Circles
LOCUS 37

Example – 30 POLE AND POLAR

Through an arbitrary fixed point P( x1 , y1 ), a variable line is drawn intersecting the circle x 2 + y 2 = a 2 at A and B
respectively. Tangents drawn to this circle at A and B intersect at Q. Find the locus of Q.

Solution :
Q (h 1 k )
B The tangents at A and B to
the circle x2 + y2 = a2
A intersect at
Q. We wish to determine
the locus of Q whose
P (x1,y1) co-ordinates we have
assumed to be (h1 k)

Fig - 32
We can view this situation from a different perspective. We have a point Q ( h, k ) from which we draw
tangents QA and QB to the circle x 2 + y 2 = a 2 . Thus, the equation of AB (which is the chord of
contact) is

T ( h, k ) = 0

⇒ hx + ky = a 2

Now, this chord of contact also passes through P ( x1 , y1 ) so that

hx1 + ky1 = a 2 ...(1)

What we have in (1) is a linear equation involving the variables h and k. Note that x1 and y1 are
constant. Thus, we can infer from (1) that ( h, k ) lies on the line

xx1 + yy1 = a 2 ...(2)

This is the required equation !


The line obtained in (2) is referred to as the polar of the point P with respect to the given circle. P is
itself referred to as the pole of the polar.
Notice that the equation of the polar can be written concisely as

T ( x1 , y1 ) = 0 : Equation of the polar for the pole P( x1 , y1 )

This example should show you that sometimes enormous simplifications are achieved using a
co-ordinate geometrical approach rather than a pure-geometrical one. Co-ordinate geometry is not
all that bad!

Mathematics / Circles
LOCUS 38

TRY YOURSELF - II

Q. 1 Three circle with radii r1 , r2 and r3 touch each other externally. The tangents at their points of contact
meet at a point whose distance from any point of contact is 4. Show geometrically that

r1r2 r3
= 16
r1 + r2 + r3

Q. 2 What is the equation of the tangent to x 2 + y 2 − 30 x + 6 y + 109 = 0 at (41 − 1) ?

Q. 3 Find the tangents to x 2 + y 2 − 6 x + 4 y − 12 = 0 parallel to 4 x + 3 y + 5 = 0.

Q. 4 Find the tangents to x 2 + y 2 = a 2 which make a triangle of area a 2 with the axes.

Q. 5 If P (a, b) and Q (b, a ) are two points (b ≠ a ), find the equation of the circle touching OP and OQ at
P and Q where O is the origin .

Q. 6 Find the equation of the normal to 3 x 2 + 3 y 2 − 4 x − 6 y = 0 at (0, 0)

Q. 7 The line 2 x − y + 1 = 0 is a tangent to a circle at (2, 5); its centre lies on x + y = 9. Find its equation.

Q. 8 If 3 x + y = 0 is a tangent to a circle whose centre is (2, –1), find the other tangent to the circle from
the origin.
Q. 9 Find the locus of the point of intersection of tangents to the circle
x 2 + y 2 + 4 x − 6 x + 9 sin 2 α + 13cos 2 α = 0 which are inclined at an angle of 2α to each other.

Q. 10 Prove that the intercept of the pair of tangents from the origin to x 2 + y 2 + 2 gx + 2 fy + k 2 = 0, on the
2λk
line y = λ is 2 .
k − g2

Mathematics / Circles
LOCUS 39

Section - 3 EQUATIONS DESCRIBING CIRCLES

Consider two circles C1 and C2 which intersect in two distinct points A and B. Our purpose is to determine the
equation of AB which will be termed the common chord of C1 and C2 .

C2
C1 AB is the common chord
A of the circles C1 and C2.
The equations of C1 and C2
are given to be S1 = 0 and
S2 = 0 respectively.
B

Fig - 33
Let the equations of the circles be

S1 ≡ x 2 + y 2 + 2 g1 x + 2 f1 y + c1 = 0

S 2 ≡ x 2 + y 2 + 2 g 2 x + 2 f 2 y + c2 = 0

Consider the equation S ≡ S1 − S2 = 0, i.e.

S ≡ (2 g1 − 2 g2 ) x + (2 f1 − 2 f 2 ) y + c1 − c2 = 0 ...(1)

Clearly, S is the equation of a straight line. Observe carefully that since A and B lie on both C1 and C2 , the co-
ordinates of A and B satisfy both S1 and S2 and thus S ≡ S1 − S2 = 0. This means that S1 − S2 = 0 represents a
straight line passing through A and B. Thus, this is precisely the common chord !

S ≡ S1 − S2 = 0 : Equation of the common chord

You are urged to observe one important fact about S. It is perpendicular to the line joining the centres of C1 and
C2 ; this is but expected.

Slope of O1O2 is
f 2 − f1
m1 =
A g2 − g1
O1 P O2
Slope of AB is from (1)
(–g1,–f1 ) (–g2,–f2 )
g 2 − g1
C1 B m2 =
f 2 − f1
Thus, m1 m2 = – 1
C2
Fig - 34

The perpendicularity of O1O2 and AB can be proved geometrically too in a straightforward manner.
Mathematics / Circles
LOCUS 40

The length of the common chord can be easily evaluated using the Pythagoras theorem:
AB = 2 O1 A2 − O1 P 2 = 2 O2 A2 − O2 P 2
where O1P and O2 P are the perpendicular distances of O1 and O2 from the common chord respectively.
Notice an interesting fact : if the length of the common chord AB is 0, it is actually the common tangent to the two
circles C1 and C2 which will touch each other externally or internally.

O1 P O2 O1 O2 P

C2
C1 C2 C1

If the two circles touch each other externally or internally (at P), then the common
chord S1 – S2 = 0 actually represents the common tangents to the two circles at P

Fig - 35

Now an interesting question arises. Suppose that C1 and C2 lie external to each other and do not intersect. What
does S1 – S2 = 0 represent in that case?

C2
C1 What does S1 – S2 = 0
represent in this case?
Note that no common
chord or common tangent
can exist.

S1= 0
S2= 0
Fig - 36
S1 − S 2 = 0 is obviously a straight line. But it is obviously not a common chord or a common tangent since these
do not exist in this case.

The answer is provided by a slight algebraic manipulation. Let a point P ( x1 , y1 ) be such that it satisfies S1 − S 2 = 0.
Thus,
S1 ( x1 , y1 ) = S2 ( x2 , y1 )

Assuming both sides are +ve 


 
⇒ S1 ( x1 , y1 ) = S 2 ( x1 , y1 )  which is true if P is external 
 to C1 and C2 . 

Mathematics / Circles
LOCUS 41

What does this equation tell us? P is a point such that the lengths of the tangents drawn from it to the two circles are
equal. Thus, any point lying on the straight line S1 − S2 = 0 will posess the property that the tangents drawn from
it to the two circles are equal. This line is termed the radical axis of the two circles.

B
A
For any point p on the
radical axis, tangents drawn
from it to C1 and C2 are of
equal length, i.e. PA = PB

C1 C2

This line is the radical axis

Fig - 37

It should be obvious that in case of intersecting (or touching) circles, the common chord (or the common tangent)
is itself the radical axis. For a situation as in Fig - 37 above, the radical axis exists but no common chord exists, For
a circle lying inside another circle, neither the radical axis nor the common chord exist :

C2
C1 C1 C2

The radical axis is the same as The radical axis is the same as the common tangent.This
the common chord. should otherwise be obvious also since for any point P on
this line, the length of tangents to both C1 and C2 is PX

C1

The radical axis is the same as the No common chord or radical axis exist
common tangent which is again abvious
Fig - 38

Mathematics / Circles
LOCUS 42

Example – 31

Show that the radical axis of three circles, whose centres are non-collinear, taken two a + a time are concurrent.
Solution: The significance of the phrase ‘non-collinear’ for the three centres should be clear to you : if the
centres are all collinear, the three radical axis will become parallel to each other instead of intersecting.

We assume the equations to the three circles to be S1 = 0, S2 = 0 and S3 = 0. The three radical axis
are therefore

R1 : S1 − S2 = 0
R2 : S2 − S3 = 0
R3 : S3 − S1 = 0
Observe that
(1) R1 + (1) R2 + (1) R3 = 0
which implies that R1 , R2 and R3 are concurrent. The point of concurrency is called the radical centre
of the three circles :
R1

C2 R1, R2 and R3 are


the three radical axis,
C1 while X, the point of
X concurrency is termed
the radical centre.
Tangents drawn from
X to the three circles
R2 will be of equal lengths
C3
R3

Fig - 39
____________________________________________________________________________________
Before proceeding we must discuss some properties of two intersecting circles; in particular, we need to understand
what we mean by the angle of intersection of two circles.

Consider two intersecting circles C1 and C2 with radii r1 and r2 respectively and centres at O1 and O2 respectively:

C1 and C2 intersect
O1 O2
at A and B.
C2
C1
B

Fig - 40
Mathematics / Circles
LOCUS 43

The angle of intersection of the two circles can be defined as the angle between the tangents to the two circles at
their point(s) of intersection, which will be the same as the angle between the two radii at the point(s) of
intersection. In particular, for example, C1 and C2 in the figure above intersect at an angle ∠O1 AO2 .

The most important case we need to consider pertaining to intersecting circles is orthogonal circles, meaning that
the angle of intersection of the two circles is a right angle. In that case, the ∠O1 AO2 . above will become a right -
angled one so that

O1 A2 + O2 A2 = O1O22 ...(1)

If the two circles and C1 and C2 have the equations

S1 : x 2 + y 2 + 2 g1 x + 2 f1 y + c1 = 0 ; r1 = g12 + f12 − c1

S2 : x 2 + y 2 + 2 g 2 x + 2 f 2 y + c2 = 0 ; r2 = g 22 + f 22 − c2

then the condition (1) becomes

r12 + r22 = O1O22

⇒ g12 + f12 − c1 + g 22 + f 22 − c2 = ( g1 − g 2 )2 + ( f1 − f 2 ) 2

⇒ 2( g1 g 2 + f1 f 2 ) = c1 + c2

Thus, the condition that the two circles given by S1 and S 2 must satisfy in order to be orthogonal is

2( g1 g 2 + f1 f 2 ) = c1 + c2 : Orthogonal circles

As an exercise, verify that the following pairs of circles intersect orthogonally :

(a) S1 : x 2 + y 2 − 6 x − 8 y + 24 = 0
S 2 : x 2 + y 2 − 8 x − 6 y + 24 = 0

(b) S1 : x 2 + y 2 + 5 x + 3 y + 7 = 0
S 2 : x 2 + y 2 − 8 x + 6 y − 18 = 0

As another example, suppose that we have to find the angle at which


S1 : x 2 + y 2 − 6 x + 4 y + 11 = 0
S2 : x 2 + y 2 − 4 x + 6 y + 9 = 0
intersect.
We have,

r1 = 2; r2 = 2; O1 ≡ (3, −2); O2 ≡ (2, −3)

Mathematics / Circles
LOCUS 44

Referring to Fig - 40, let the angle of intersection, ∠O1 AO2 be θ . By the cosine rule in ∆O1 AO2 , we have

r12 + r22 − O1O22 1


cos θ = =
2r1r2 2

π
⇒ θ=
4

π
Thus, the two circles intersect at an angle equal to .
4

Example – 32

A circle C passes through (1, 1) and cuts the following two circles orthogonally :

S1 : x 2 + y 2 − 8 x − 2 y + 16 = 0
S2 : x 2 + y 2 − 4 x − 4 y − 1 = 0

Find the equation of C.


Solution: We assume the equation of C to be

S : x 2 + y 2 + 2 gx + 2 fy + c = 0

Applying the condition of orthogonality of S with S1 and S2 , we obtain :

with S1 : 2( −4 g − f ) = c + 16

⇒ 8 g + 2 f + c = −16 ...(1)

with S 2 : 2( −2 g − 2 f ) = c − 1

⇒ 4g + 4 f + c = 1 ...(2)
The third condition can be obtained using the fact that C passes through (1, 1):

12 + 12 + 2 g (1) + 2 f (1) + c = 0

⇒ 2 g + 2 f + c = −2 ...(3)
Solving (1), (2) and (3), we obtain
7 23
g = − , f = , c = −5
3 6
Thus, the equation of the circle C is

S : 3x 2 + 3 y 2 − 14 x + 23 y − 15 = 0

Mathematics / Circles
LOCUS 45

Example – 33

Prove that the locus of the centres of the circles cutting two given circles orthogonally is their radical axis.
Solution: This assertion means that any circle cutting two given circles orthogonally will have its centre lying on
the radical axis of the two given circles.
Assume the two fixed given circles to have the following equations:

S1 : x 2 + y 2 + 2 g1 x + 2 f1 y + c1 = 0

S 2 : x 2 + y 2 + 2 g 2 x + 2 f 2 y + c2 = 0

Let the variable circle be S : x 2 + y 2 + 2 gx + 2 fy + c = 0 so that its centre is ( − g , − f ) whose locus


we wish to determine.
Applying the condition for orthogonality, we obtain

2( gg1 + ff1 ) = c + c1 ...(1)

2( gg2 + ff 2 ) = c + c2 ...(2)

By (1) – (2), we obtain

2 g ( g1 − g2 ) + 2 f ( f1 − f 2 ) = c1 − c2

Using ( x, y ) instead of ( − g , − f ), we obtain the locus as 2 x( g1 − g 2 ) + 2 y ( f1 − f 2 ) + (c1 − c2 ) = 0


which is the same as the equation of the radical axis, i.e. S1 − S2 = 0.

We can also prove this assertion of this question in a very straightforward manner using a pure geometric
approach. Let C be a circle intersecting the two given circles C1 and C2 orthogonally as show in the
figure below:

C intersects C1 and
O C2 orthogonally.
Thus,
A B OA ⊥ O1 A and
OB ⊥ O2 B

O1 O2
C1
C2

Fig - 41

OA ⊥ O1 A and OB ⊥ O2 B implies that OA and OB are simply the tangents drawn from O to C1 and
C2 . Since OA and OB are the radii of the same circle C, we have OA = OB.
Mathematics / Circles
LOCUS 46

Thus, O is a point such that tangents drawn from it to the two circles C1 and C2 are equal in length,
which implies that O lies on the radical axis of C1 and C2 .

From this property, a straightforward corollary follows. For three fixed circles, a circle with centre at
the radical centre and radius equal to the length of the tangent from it to any of the circles will intersect
all the three circles orthogonally. As an exercise, find the equation to the circle C cutting the following
three circles orthogonally :

C1 : x 2 + y 2 − 2 x + 3 y − 7 = 0

C2 : x 2 + y 2 + 5 x − 5 y + 9 = 0

C3 : x 2 + y 2 + 7 x − 9 y + 29 = 0

Example – 34

Prove that two circles, both of which pass through the point (0, a) and (0, − a ) and touch the line y = mx + c, will
cut orthogonally if c 2 = a 2 (2 + m2 )

Solution: It should be clear that the y-axis is the common chord of the two circles C1 and C2 :

(01,a) y - axis is the common


chord of C1 and C2
Also, the centres of
x both C1 and C2 with lie
(0,-a) on the x - axis
C1
C2

Fig - 42

Let the centres of C1 and C2 be (− g1 , 0) and (− g 2 , 0) so that their radii become g12 + a 2 and

g 22 + a 2 respectively. Their equations then become :

C1 : x 2 + y 2 + 2 g1 x − a 2 = 0

C2 : x 2 + y 2 + 2 g 2 x − a 2 = 0

Mathematics / Circles
LOCUS 47

The line y = mx + c touches both C1 and C2 so that the perpendicular distance of the centres of C1
and C2 from this line must be respectively equal to their radii. Thus, we obtain

mg1 − c mg 2 − c
= g12 + a 2 and = g 22 + a 2
1 + m2 1+ m 2

⇒ g12 + 2mcg1 + a 2 (1 + m 2 ) − c 2 = 0

and g 22 + 2mcg 2 + a 2 (1 + m 2 ) − c 2 = 0

Thus, g1 and g2 are the roots of the equation

g 2 + 2mcg + a 2 (1 + m 2 ) − c 2 = 0
so that

g1 g 2 = a 2 (1 + m2 ) − c 2

Finally, C1 and C2 are orthogonal if the condition

2( g1 g 2 + f1 f 2 ) = c1 + c2
is satisfied, i.e.

2{(a 2 (1 + m2 ) − c 2 ) + (0)(0)} = −2a 2

⇒ c 2 = a 2 (2 + m 2 )

This is the required relation for C1 and C2 to be orthogonal.

Example – 35

Consider the following two circles:

S1 : x 2 + y 2 − 16 = 0
S2 : x 2 + y 2 − 8 x − 12 y + 16 = 0

Circles are drawn which are orthogonal to both these circles. Tangents are drawn from the centre of the variable
circle to S1 . Find the mid locus of the mid-point of the chord of contact so formed.

Solution: The centre of the variable circle, say ( x1 , y1 ), lies on the radical axis of the two given circles, i.e. on
S1 − S2 = 0 by the result obtained in Example - 33.

Mathematics / Circles
LOCUS 48

Thus, ( x1 , y1 ) must satisfy

S1 − S2 = 0 or 8 x + 12 y − 32 = 0

⇒ 8x1 + 12 y1 − 32 = 0 ...(1)

From ( x1 , y1 ), two tangents are drawn to S1. The equation of the chord of contact is therefore

T ( x1 , y1 ) = 0

⇒ xx1 + yy1 = 16 ...(2)

Let the mid-point of the chord of contact so formed be M ( h, k ). We can also write the equation of
the same chord of contact using the equation we derived for a chord bisected at a given point.
Thus, the chord of contact can also be represented by the equation

T ( h, k ) = S ( h, k )

⇒ hx + ky = h 2 + k 2 ...(3)

Since (2) and (3) are the same lines, we have

x1 y1 16
= = 2
h k h + k2

16h 16k
⇒ x1 = ; y1 = 2 ...(4)
h +k
2 2
h + k2

Using (4) in (1), we finally obtain a relation in ( h, k ) :

8 x1 + 12 y1 − 32 = 0

128h 192k
⇒ + 2 = 32
h +k
2 2
h + k2

⇒ h 2 + k 2 − 4h − 6k = 0

Using ( x, y ) instead of ( h, k ) we obtain the required locus as

x2 + y2 − 4 x − 6 y = 0

Mathematics / Circles
LOCUS 49

TRY YOURSELF - III

Q. 1 Find the equation of the circle which intersects x 2 + y 2 − 6 x + 4 y − 3 = 0 orthogonally, passes through
(3, 0) and touches the y-axis.
Q. 2 At what angle do the circles

S1 : x 2 + y 2 − 4 x + 6 y + 11 = 0
S 2 : x 2 + y 2 − 2 x + 8 y + 13 = 0

intersect ?
Q. 3 Find the co-ordinates of the point from which the tangents drawn to the following three circles are of
equal lengths :

S1 : 3x 2 + 3 y 2 + 4 x − 6 y − 1 = 0
S2 : 2 x 2 + 2 y 2 − 3x − 2 y − 4 = 0
S3 : 2 x 2 + 2 y 2 − x + y − 1 = 0

Q. 4 Find the locus of the centre of the circle passing through ( a, b) and orthogonal to x 2 + y 2 = k 2 .

Q. 5 Let A(a1 , b1 ) and B(a2 , b2 ) be two fixed points and O be the origin. Circles are drawn on OA and
a1b2 − a2b1
OB as diameters. Prove that the length of the common chord is .
AB

Mathematics / Circles
LOCUS 50

Section - 4 FAMILY OF CIRCLES

By a family of circles, we will mean a set of circles satisfying some given property (or properties). For example, the
family of circles with each circle having its centre lying in the first quadrant and touching both the co-ordinate axes
can be represented by the equation

( x − a) 2 + ( y − a )2 = a 2 ...(1)

where a is a positive real number.


The important point to observe is that a is a variable here. As we vary a, we get different circles belonging to this
family, but due to the constraint imposed by (1), all circles of this family satisfy the specified property.
y

Some members of the family of


circles given by (1).
The centre of a circle is (a, a) and its
radius is a, where a is a real
positive variable. As a is varied,
we obtain different members in
this family

x
Fig - 43
We intend to discuss in this section certain families that are of significant importance. In all cases, the family will be
represented by an equation containing a real variable, which when varied will give rise to different members of this
family.

FAMILY OF CIRCLES PASSING THROUGH THE INTERSECTION POINTS OF


TYPE 1: A GIVEN CIRCLE AND A GIVEN LINE

We are given a fixed circle and a fixed line with equations S = 0 and L = 0 respectively. We want to
find out the equation of the family of the circles passing through the points of intersection of S = 0 and
L =0.

A
The circles S = 0 and L = 0
are fixed. The dotted circles
represent some of the members
of the family of circles passing
through the intersection points
A and B of S = 0 and L = 0

B
S=0

L=0
Fig - 44

Mathematics / Circles
LOCUS 51

By now, it should be apparent to you how to specify the equation of this family in terms of a real
variable. Any circle F belonging to this family can be written as

F ≡ S + λ L = 0 where λ ∈ ! .

The truth of this assertion can be easily verified. F as defined from this equation is definitely a circle
since it satisfies the required constraints for its equation to be a circle. To see this, let

S : x 2 + y 2 + 2 gx + 2 fy + c = 0

L : px + qy + r = 0
We then have

F : S + λL = 0

⇒ x 2 + y 2 + (2 g + λ p) x + (2 f + λ q) y + c + λ r = 0
which is definitely the equation of a circle. To see that F passes through A and B, note that since A and
B satisfy both the equations S = 0 and L = 0, they will obviously also satisfy the equation S + λ L = 0.

Thus, F ≡ S + λ L = 0 is the required family of circles. As we vary λ , we will get different circles
belonging to this family. In particular, note that for λ = 0, we get the circle S = 0 itself, while in the
limit λ → ∞, we get the ‘circle’ L = 0, which is actually a line but can be considered a circle with
centre lying at infinity and an infinitely large radius.

Example – 36

Write the equation for

(a) the family of circles passing through ( x1 , y1 ) and ( x2 , y2 )

(b) the family of circles touching the line L = 0 at ( x1 , y1 )

Solution (a) Till now we have seen how to write the equation for a family of circles passing through the intersection
points of a circle and a line. Thus, in the current example, we first define a fixed circle passing
through ( x1 , y1 ) and ( x2 , y2 ) and the line through these two points.

The fixed circle S can easily be taken to be the one with ( x1 , y1 ) and ( x2 , y2 ) as the end points of
a diameter :

S : ( x − x1 )( x − x2 ) + ( y − y1 )( y − y2 ) = 0
The fixed line can be written using the two-point form :
x y 1
y − y1 x − x1
L: = ⇒ L : x1 y1 1 = 0
y2 − y1 x2 − x1
x2 y2 1
Mathematics / Circles
LOCUS 52

S=0 Given two fixed points


(x1, y1) and (x2, y2),
we define the fixed circle
as the one with these points
being the end-points of a
L=0 diameter. The fixed line is
(x 1 , y1 ) (x2, y2)
simply the line passing
through these two points.

Fig - 45
We can now write the required family of circles as :

F : S + λL = 0

⇒ ( x − x1 )( x − x2 ) + ( y − y2 )( y − y2 ) + λ L = 0 ...(1)

where L = 0 represents the line through the two given points

(b) We now want the family of circles to be such that each circle touches L = 0 at ( x1 , y1 ) .

We could evaluate it by letting x2 → x1 and y2 → y1 in (1). In this limit, L = 0 will simply


become the tangent to any member of the family F given by (1). (Convince yourself about this
point.
Thus, the required family is

F ′ : ( x − x1 ) 2 + ( y − y1 ) 2 + λ L = 0

TYPE 2: FAMILY OF CIRCLES TOUCHING A GIVEN CIRCLE AT A GIVEN POINT

Let the equation of the fixed circle be

S : x 2 + y 2 + 2 gx + 2 fy + c = 0

and let there be a point P( x1 , y1 ) lying on this circle. We wish to determine the equation of the family
of circles touching S at P.

The dotted circles are


some of the members of
P the family of circles in
(x1, y1) which each circle touches
S at P
S=0

Fig - 46

Mathematics / Circles
LOCUS 53

We can write the equation of the tangent to S = 0 at P as

T : xx1 + yy1 + g ( x + x1 ) + f ( y + y1 ) + c = 0

Once we have a circle ( S = 0) and a line (T = 0) intersecting or touching the circle, we can write the
equation of the family of circles passing through the point (s) of intersection of the circle and the line,
using the result derived in the last article. Thus, the required family can be represent as

F : S + λT = 0

⇒ F : x 2 + y 2 + 2 gx + 2 fy + c + λ ( xx1 + yy1 + g ( x + x1 ) + f ( y + y1 ) + c = 0

⇒ F : x 2 + y 2 + (2 g + λ x1 + λ g ) x + (2 f + λ y1 + λ f ) y + c + λ gx1 + λ fy1 + λ c = 0

As we vary λ , we will obtain different members belonging to this family.

FAMILY OF CIRCLES PASSING THROUGH THE INTERSECTION POINT(S) OF


TYPE 3: TWO GIVEN CIRCLES

Let the two fixed circles be

S1 : x 2 + y 2 + 2 g1 x + 2 f1 y + c1 = 0

S 2 : x 2 + y 2 + 2 g 2 x + 2 f 2 y + c2 = 0

and their points of intersection be A( x1 , y1 ) and B( x2 , y2 ). In case the two circles touch each other,
A and B will be the same.
We wish to determine the family of circles passing through A and B.

A
The dotted circles represent
some members of the family
of circles in which each member
S1 = 0 passes through A and B

B S2 = 0

Fig - 47
You might be able to extrapolate from the last few cases that the equation representing this family
will be
F : S1 + λ S 2 = 0

Mathematics / Circles
LOCUS 54

You can verify this by writing the equation for F in standard form and observing that it does indeed
represent a circle. Also, since (the co-ordinates of) A and B satisfy both S1 = 0 and S2 = 0, they
have to satisfy the equation for F. Note one important point: λ cannot be equal to –1 otherwise F will
become the common chord of S1 = 0 and S2 = 0 instead of representing a circle.

Example – 37
Find the equation of the circle which passes through the points of intersection of the circles

S1 : x 2 + y 2 − 6 x + 2 y + 4 = 0

S2 : x 2 + y 2 + 2 x − 4 y − 6 = 0

and whose centre lies on the line y = x.

Solution: Let the required equation be S = 0. Then, by the previous article, we can find some λ ∈ ! and
λ ≠ −1 such that

S ≡ S1 + λS 2 = 0

⇒ S ≡ (1 + λ ) x 2 + (1 + λ) y 2 + (2λ − 6) x + (2 − 4λ) y + 4 − 6λ = 0 ...(1)

The centre of S from this equation comes out to be

 (λ − 3) (1 − 2λ) 
centre ≡  − ,− 
 1+ λ 1+ λ 

Since the centre lies on the line y = x, we have

(λ − 3) (1 − 2λ )
− =−
1+ λ 1+ λ
4
⇒ λ=
3
We now substitute this value back in (1) to obtain the equation for S:

7 2 7 2 10 10
S: x + y − x− y−4 = 0
3 3 3 3

⇒ S : x 2 + y 2 − 10 x − 10 y − 12 = 0

Example – 38
A family of circles passing through the points A (3, 7) and B (6, 5) cuts the circle x 2 + y 2 − 4 x − 6 y − 3 = 0. Show
that the common chord of the fixed circle and the variable circle(belonging to the family) will always pass through
a fixed point. Find that point.
Mathematics / Circles
LOCUS 55

Solution: We can write the equation to the specified family of circles by first writing the equation L of the line AB:

y−7 2
L: =
x − 3 −3

⇒ L : 2 x + 3 y = 27
The required family can now be written as

F : ( x − 3)( x − 6) + ( y − 7)( y − 5) + λL = 0 where λ ∈ !

⇒ F : x 2 + y 2 + (2λ − 9) x + (3λ − 12) y + (53 − 27λ ) = 0


The common chord of F and the given circle S is :

S−F =0
⇒ (5 − 2λ ) x + (6 − 3λ ) y + (27λ − 56) = 0

⇒ (5 x + 6 y − 56) − λ (2 x + 3 y − 27) = 0

⇒ L1 + µL2 = 0

That the common chord can be written like L1 + µL2 = 0 implies that it will always pass through the
intersection point of L1 and L2, what ever the value of µ may be. This intersection point can be
 23 
obtained (by simultaneously solving L1 and L2) to be  2, .
 3 

Example – 39

Find the equation of the circle which touches the line x − y = 0 at the origin and bisects the circumference of the
circle x 2 + y 2 + 2 y − 3 = 0 .

Solution: In example -36, we evaluated the equation of the family of circles all touching a given line at a given
point. Here, the given line is x − y = 0 and the given point is (0, 0). Thus, the equation of the family is

F : ( x − 0) 2 + ( y − 0) 2 + λ( x − y ) = 0

⇒ F : x 2 + y 2 + λ x − λy = 0 ...(1)

We need to find the value of λ for which the circle in (1) bisects the circumference of the given circle
S : x 2 + y 2 + 2 y − 3 = 0, which means that the common chord of the required circle and S will the
diameter of S.

Mathematics / Circles
LOCUS 56

A
If the variable circle
bisects the circumference of S,
this mean that the common
chord AB must be the diameter
of S.
S=0
B
The variable
circle

Fig - 48
The common chord AB is

F −S =0
⇒ λx − (λ + 2) y + 3 = 0
Since this is the diameter of S, the centre of S, i.e. (0, –1), must lie on it (satisfy its equation). Thus, we
obtain λ as

λ (0) − (λ + 2)( −1) + 3 = 0

⇒ λ = −5
Finally, we substitute this value of λ back in (1) to get the required equation of the circle as

x 2 + y 2 − 5 x + 5 y = 0.

Example – 40

Two circles, each of radius 5 units, touch each other at (1, 2). If the equation of their common tangent is
4 x + 3 y = 10, find the equation of the circles.
Solution: The following diagram explains the situation better:

S2 = 0

S1 = 0 The two circles


(1, 2) touch each
C1 C2 other at (1, 2).

4x + 3y = 10

Fig - 49
We describe here two alternatives that can be used to solve this question

Mathematics / Circles
LOCUS 57

Alternative - 1: Find the centres of the two circles

4 3
The slope of the common tangent is − . Therefore, the slope of C1C2 is , i.e.
3 4
3
m = tan θ =
4

3 4
⇒ sin θ = and cos θ =
5 5
Using the polar form, we can write the co-ordinates of any point on the line C1C2:
x −1 y − 2
= =r
cos θ sin θ
⇒ x = 1 + r cos θ, y = 2 + r sin θ

Substituting r = ± 5 gives the two centres as should be apparent from the figure.
Thus, the two centres are

C1 ≡ (5, 5) and C2 ≡ (−3, − 1)

Thus, the two equations are

S1 : x 2 + y 2 − 10 x − 10 y + 25 = 0

S 2 : x 2 + y 2 + 6 x + 2 y − 15 = 0

Alternative - 2: Use a family of circles approache

The family of circles touching L = 0 at ( x1 , y1 ) can be written, as described


earlier, as

( x − x1 ) 2 + ( y − y1 ) 2 + λL = 0
In the current case, this becomes

( x − 1) 2 + ( y − 2) 2 + λ (4 x + 3 y − 10) = 0

⇒ x 2 + y 2 + (4λ − 2) x + (3λ − 4) y + (5 − 10λ ) = 0 ...(1)


The radius of the required circle is 5. Thus,
2
 3λ − 4 
(2λ − 1) + 
2
 − (5 − 10λ ) = 5
2

 2 
This is a quadratic in λ which gives two values of λ :

λ = ±2
Using these values in (1), we obtained the two required circles.

Mathematics / Circles
LOCUS 58

TRY YOURSELF - IV

Q. 1 Find the equations of the circles with radius 4 and passing through the points of intersection of

S1 : x 2 + y 2 − 2 x − 4 y − 4 = 0

S 2 : x 2 + y 2 − 10 x − 12 y + 40 = 0

Q. 2 Find the equation of the circle whose diameter is the common chord of the circles ( x − a) 2 + y 2 = a 2
and x 2 + ( y − b)2 = b 2 .

Q. 3 Find the equation of the circle passing through (2, 1) and touching the line x + 2 y = 1 at (3, –1).

Q. 4 Tangents are drawn from the origin to x 2 + y 2 + 6 x + 4 y − 12 = 0. Find the equation of the circle
passing through the points of contact of these tangents and the origin.

Q. 5 Find the equation of a circle which touches the line x + y = 5 at the points (–2, 7) and cuts the circle
x 2 + y 2 + 4 x − 6 y + 9 = 0 orthogonally.

Mathematics / Circles
LOCUS 59

SOLVED EXAMPLES

Example – 01

The circle x 2 + y 2 − 4 x − 4 y + 4 = 0 is inscribed in a triangle which has two of its sides along the co-ordinate
axes. If the locus of the circumcentre of the triangle is

x + y − xy + k x 2 + y 2 = 0

find the value of k.


Solution : The situation is described clearly in the figure below:
y

B
(0, b) The circle S = 0
is fixed. The line is variable,
intersecting the axis in A(a, 0)
and B (0, b) respectively.
(2,2) We are concerned with the locus
of the circumcentre of ∆ OAB.
S= 0 A
x
0 (a, 0) L
Fig - 50
The equation of L is, using the intercept form,
x y
L: + =1
a b
The distance of the centre of S, i.e. (2, 2) from L must equal the radius of S which is 2. Thus,

 We now use the fact that 


2 2  L(2, 2) is negative since 
+ −1
a b  
=2 (2, 2) and the origin lie 
1 1 on the same side of L and 
+  L(0, 0) is negative 
a2 b2  

⇒ 2a + 2b − ab + 2 a 2 + b 2 = 0 ...(1)

From pure geometric considerations, the circumcentre C of ∆OAB lies on AB and is in fact, the
mid-point of AB.
Thus,

a b
C ≡ , 
 2 2

Mathematics / Circles
LOCUS 60

Slightly manipulating (1), we obtains


2 2
a b  a  b  a b
+ −    +   +   = 0 ...(2)
2 2  2  2  2 2

a b a b
The locus of  ,  is given by (2). Using ( x, y ) instead of  ,  , we obtain
 2 2 2 2

x + y − xy + x 2 + y 2 = 0 ...(3)

Upon comparing (3) with the locus specified in the question, we obtain k = 1.

Example – 02

Consider a curve ax 2 + 2hxy + by 2 = 1 and a point P not on the curve. A line drawn from P intersects the curve at
points Q and R. If the product PQ ⋅ PR is independent of the slope of the line, then show that the curve is a circle.
Solution : Since distances are involved from a fixed point, it would be a good idea to use the polar form of the
line to write the co-ordinates of Q and R.

Let P be ( x1 , y1 ) and let θ denote the slope of the variable line. For any point ( x, y ) lying on this line
at a distance r from P, we have

x − x1 y − y1
= =r
cos θ sin θ

⇒ x = x1 + r cos θ ; y = y1 + r sin θ

If ( x, y ) lies on the given curve, it must satisfy the equation of the curve :

a ( x1 + r cos θ ) 2 + b( y1 + r sin θ ) 2 + 2h( x1 + r cos θ )( y1 + r sin θ ) = 1

⇒ {a cos 2 θ + b sin 2 θ + h sin 2θ }r 2 + 2{ax1 cos θ + by1 sin θ + h( x1 cos θ

+ y1 sin θ )}r + ax12 + by12 + 2 hx1 y1 − 1 = 0 ...(1)

This quadratic has two roots in r, say r1 and r2 , which will actually correspond to PQ and PR
since Q and R lie on the curve. Thus, PQ ⋅ PR being independent of θ means that r1r2 for (1) is
independent of θ i.e.

ax12 + by12 + 2hx1 y1 − 1


is independent of θ
a cos 2 θ + b sin 2 θ + h sin 2θ

ax12 + by12 + 2hx1 y1 − 1


⇒ is independent of θ
 a +b   a −b 
 +
   cos 2θ + h sin 2θ
 2   2 
Mathematics / Circles
LOCUS 61

 1 + cos 2θ 1 − cos 2θ 
The denominator was obtained in this form by writing cos θ as and sin 2 θ as
2

 2 2 

ax12 + by12 + 2hx1 y1 − 1


⇒ is independent of θ
2
 a+b  a −b 
+   + h {sin(2θ + φ}
2

 2   2 
  a −b  
The denominator was obtained by combining the two trignometric terms.tan φ is   
  2h  

2
a−b 
⇒ This is only possible when   +h =0
2

 2 

⇒ a = b and h = 0
Thus, the equation of the given curve reduces to

ax 2 + ay 2 = 1

which is clearly the equation of a circle.

Example – 03

The equation of a circle is S : 2 x ( x − a ) + y (2 y − b) = 0 where a, b ≠ 0. Find the condition on a and b if two


 b
chords each bisected by the x-axis, can be drawn to the circle from the point P  a,  .
 2
a b
Solution: Observe that the centre of S is  ,  and P lies on the circle:
 2 4

b The centre of the circle


P (a , )
2 S is ( a , b );
a,b 2 4
( ) the point P lies on the circle.
2 4
x In fact, OP is a diameter
O
of the circle

Fig - 51

Mathematics / Circles
LOCUS 62

Let us consider a chord PQ of the circle bisected at the x-axis, say, at the point ( h, 0). We can write
the equation of PQ as :
T ( h, 0) = S ( h, 0)

where S ( x, y ) is b
x 2 + y 2 − ax − y=0
2
a b
⇒ hx − ( x + h) − ( y + 0) = h 2 − ah
2 4

 a b  ah 2
⇒ h − x − y + −h  = 0 ...(1)
 2 4  2 

 b
Since this chord passes through P  a,  , the co-ordinates of (P) must satisfy (1). Thus,
 2

 a b  b  ah
h− a −  + − h2 = 0
 2 42 2

3ah a 2 b 2
⇒ − − − h2 = 0
2 2 8

3ah a 2 b 2
⇒ h −
2
+ + =0 ...(2)
2 2 8
We want to have two such possible chords PQ bisected at the x-axis. Thus, we must have two
distinct values of h which can happen if the discriminant of (2) is positive. Thus,

9a 2  a2 b2 
> 4 + 
4  2 8

⇒ a 2 > 2b 2
This is the required condition that a and b must satisfy.

Example – 04

Let T1 and T2 be two tangents drawn from (–2, 0) to the circle x 2 + y 2 = 1. Determine the circles touching C and
having T1 and T2 as their pair of tangents. Also find all the possible pair wise common tangents to these circles.

Mathematics / Circles
LOCUS 63

Solution: Note that two such circles can be drawn, as shown in the figure below :

y R
C1 and C2 are the two
possible circles,
with centres
Q (−x1, 0) and (x2, 0)
P
S x
(-2, 0) A O B(x2 ,0)
C1
Centre at
( − x1 ,0 )
x2 + y2 = 1

R'
C2

Fig - 52

Let the radii of C1 and C2 be r1 and r2 respectively. We can evaluate the coordinates of both A and
B very easily from geometrical considerations. We have

∆ASP ∼ ∆OSQ ∼ ∆BSR

AS OS BS
⇒ = =
AP OQ BR

Now, AS = 2 − x1 , AP = r1 , OS = 2, OQ = 1, BS = 2 + x2 , BR = r2

2 − x1 2 2 + x2
⇒ = =
r1 1 r2

Also, note that x1 = 1 + r1 and x2 = 1 + r2 . Thus,

2 − x1 2 + x2
⇒ =2=
x1 − 1 x2 − 1
4
⇒ x1 = and x2 = 4
3
1
⇒ r1 = and r2 = 3
3

Mathematics / Circles
LOCUS 64

Thus, the equations to C1 and C2 are


2 2
 4 1
C1 :  x +  + y 2 =  
 3 3
C2 : ( x − 4) 2 + y 2 = 32

Now we determine all the possible common tangents to C1 and C2 .


Two such possible tangents are simply SR and SR′ which both touch C1 and C2 on the same side.
There will be two other possible common tangents, each having A and B on the opposite side of it as
shown in the figure below :

These tangents are


P termed the
X transverse tangents
A θ x to C1 and C2.
B
C1

Q
C2

Fig - 53

Let X have the co-ordinates ( x, 0). Note that ∆APX ∼ ∆BQX .


Thus,
AX BX
=
AP BQ
4
x+
⇒ 3 = 4− x
1/ 3 3
4
⇒ x=−
5
Also,
AP 13 5
sin θ = = =
AX −4 5 + 4 3 8
5 5
⇒ tan θ = =
8 −5
2 2
39
−4 5
Thus, the two transverse tangents pass through  , 0  and have slopes ± . Their equations
 5  39
are therefore
5  4
y=± x+ 
39  5
Mathematics / Circles
LOCUS 65

Example – 05
 1 + 2a 1 − 2a 
Find the values of a for which the line x + y = 0 bisects two chords drawn from a point P  ,  to
 2 2 

 1 + 2a   1 − 2a 
the circle S : x 2 + y 2 −   x −   y = 0 .
 2   2 

Solution: This question is somewhat similar to Example -3. Note that the point P lies on S.

Since the mid-point M of the chord(s) drawn from P lies on x + y = 0, we can assume its co-ordinates
to be ( h, − h). The equation of the chord of S bisected at M then becomes:

T ( h, − h ) = S ( h, − h )

λ λ
⇒ hx − hy − ( x + h) − ( y − h ) = 2 h 2 − λ h + λ h ...(1)
2 2

1 + 2a 1 − 2a
where the substitution λ = ,λ= has been done for convenience.
2 2

Since the chord (1) passes through P(λ , λ ), its co-ordinates must satisfy (1). Thus,

λ λ
hλ − hλ − ( λ + h ) − ( λ − h ) = 2h 2 − λ h + λ h
2 2

⇒ 4h 2 − 3(λ − λ )h + (λ 2 + λ 2 ) = 0 ...(2)

Since we want two possible chords to exist satisfying the given property, the quadratic in (2) must
yield distinct values of h. Thus, its discriminant must be positive :

9(λ − λ ) 2 > 16(λ 2 + λ 2 )

Substituting the values of λ and λ and simplifying, we obtain

a2 − 4 > 0

⇒ a ∈ ( −∞, −2) ∪ (2, ∞ )

These are the required values of a.

Example – 06
S1 S 2
If S1 = 0 and S2 = 0 are circles having radii r1 and r2 respectively, prove that the circles S : + = 0 and
r1 r2
S1 S 2
S ': − = 0 intersect orthogonally.
r1 r2

Mathematics / Circles
LOCUS 66

Solution: Let the equation of the two given circles be

S1 : x 2 + y 2 + 2 g1 x + 2 f1 y + c1 = 0 ; r12 = g12 + f12 − c1

S 2 : x 2 + y 2 + 2 g 2 x + 2 f 2 y + c2 = 0 ; r22 = g 22 + f 22 − c2
Then, the equation of S and S ' are

1 1 g g   f f  c c 
S : ( x2 + y2 )  +  + 2  1 + 2  x + 2  1 + 2  y +  1 + 2  = 0
 r1 r2   r1 r2   r1 r2   r1 r2 
1 1 g g  f f  c c 
S ′ : ( x2 + y2 )  −  + 2  1 − 2  x + 2  1 − 2  y +  1 − 2  = 0
 r1 r2   r1 r2   r1 r2   r1 r2 
We now check the orthogonality condition for S and S', i.e. we check whether
2( g S g S ' + f S f S ' ) = cS + cS '

Thus,

 g1 g 2  g1 g 2   f1 f 2  f1 f 2 
 +  −   +  − 
2( g S g S ' + f S f S ' ) = 2  r1 r2  r1 r2 
+2 1
r r2  r1 r2 
 1 1  1 1   1 1  1 1 
 +  −   +  − 
 r1 r2  r1 r2   r1 r2   r1 r2 

 g12 + f12 g 22 + f 22 
 − 
 r12 r22 
=2
1 1
 2− 2
 r1 r2 
 c1 c2 
 2− 2
= 2 1
r r2 
1 1 ...(1)
 2− 2
 r1 r2 

c1 c2 c1 c2
+ −
r1 r2 r1 r2
whereas cS + cS ' = +
1 1 1 1
+ −
r1 r2 r1 r2

 c1 c2 
 2− 2
= 2 1
r r2 
1 1 ... (2)
 2− 2
 r1 r2 
The equality of (1) and (2) implies that the two circles are orthogonal.

Mathematics / Circles
LOCUS 67

Example – 07

Let S1 = 0 and S2 = 0 be two circles with radii r1 and r2 respectively. Find the locus of the point P at which the

two circles subtend equal angles.

Solution: Consider S1 = 0 which subtends an angle α (say) at P (h, k )

(-g,-f ) P ( h , k)
C1

S1= 0
Fig - 54
We have,

C1T = r1 and PT = S1 (h, k )

Thus, in ∆C1 PT , we have

α r1
tan = ...(1)
2 S1 (h, k )

We can write an exactly analogous equation for S 2 which subtends the same angle α at P:

α r2
tan = ...(2)
2 S 2 (h, k )

From (1) and (2), we have


r1 r2
=
S1 (h, k ) S 2 ( h, k )

Using ( x, y ) instead of ( h, k ), we obtain the required locus as

S1 ( x, y ) S 2 ( x, y )
=
r12 r22

Example – 08

Circles are drawn passing through the origin O to intersect the co-ordinate axes at points P and Q such that
m ⋅ OP + n ⋅ OQ is a constant. Show that the circles pass through a fixed point other than the origin.

Mathematics / Circles
LOCUS 68

Solution: Consider one such circle as shown in the figure below:


y

Q
(0,q)
Given that
mOP + nOQ = k (a constant)
C we need to show that
S will always pass through
S
a fixed point

x
O P(p, 0)

Fig - 55

We have OP = p and OQ = q. Thus,

mp + nq = k ...(1)

 p q
Now, observe that the centre C of the circle will be  ,  and its radius will be OC. Thus, the
 2 2
equation of the circle is
2 2
 p  q p2 q2
x−  + y−  = +
 2  2 4 4

⇒ x 2 + y 2 − px − qy = 0 ...(2)

From (1), we can find q in terms of p and substitute in (2) so that (2) becomes

 mp − k 
x 2 + y 2 − px +  y=0
 n 

⇒ {n( x 2 + y 2 ) − ky} + p{− nx + my} = 0


We see that the variable circle can be written as

S0 + pL0 = 0, p ∈ !

where S0 ≡ n( x 2 + y 2 ) − ky = 0 and L0 ≡ − nx + my = 0 . Thus, the variable circle will always pass


through the two intersection points of S0 and L0, one of them obviously being the origin. The other
fixed point can be obtained by simultaneously solving the equations of S0 and L0 to be

 mk nk 
 2 , 2 2 
m +n m +n 
2

Mathematics / Circles
LOCUS 69

Example – 09

The two curves

S1 : ax 2 + 2hxy + by 2 − 2 gx − 2 fy + c = 0

S2 : a′x 2 − 2hxy + ( a′ + a − b) y 2 − 2 g ′x − 2 f y′ + c = 0

 g′ + g f ′ + f 
intersect at four concyclic points A, B , C and D. Let P be the point  ′ ,  . Find the value
 a + a a′ + a 
PA2 + PB 2 + PC 2
of .
PD 2

Solution: From what we’ve learnt in this chapter, we can write the equation of an arbitrary curve S passing
through the intersection points of two given curves, here S1 and S 2 , as

S ≡ S1 + λ S2 = 0 where λ ∈ !
Here,

S : ( a + λ a′) x 2 + 2h(1 − λ ) xy + (b + λ (a′ + a − b)) y 2 − 2 x( g + g ′λ ) − 2 y ( f + f ′) + c (1 + λ ) = 0 ..(1)

Since this curve S passes through the four concyclic points A, B , C , D its equation in (1) must be
that of a circle. Thus, we impose the necessary constraints :

Coeff. of x 2 = Coeff. of y 2 ⇒ a + λ a′ = b + λ ( a′ + a − b )

⇒ λ =1
Coeff. of xy = 0 ⇒ λ =1

Thus, for λ = 1, the equation in (1) represents a circle. This equation now becomes :

S : (a + a′) x 2 + ( a + a′) y 2 − 2( g + g ′) x − 2( f + f ′) y + 2c = 0

 g + g′ f + f ′ 
The centre of S therefore is  ,  . But this is the same as the point P!. Thus, P is the
 a + a′ a + a′ 
centre of the circle passing through A, B , C , D so that

PA = PB = PC = PD
Thus, we simply have

PA2 + PB 2 + PC 2
=3
PD 2

Mathematics / Circles
LOCUS 70

Example – 10

Let x, y and z be the lengths of the perpendiculars dropped from the circumcentre in a triangle ABC to the sides
BC, CA, and AB respectively. The lengths of BC, CA and AB are a, b and c respectively. Show that

a b c abc
+ + =
x y z 4 xyz

Solution: We can attempt this question through a co-ordinate approach but the nature of the problem posed
suggests that a plane geometric approach would be better since no equations need to be determined
here.
Refer to the following diagram which shows the situation clearly :
A

The circumcentre is O.
b c All lengths have been
y
O z marked.
Note that
θ COB = 2θ = 2 A
x
⇒θ= A
P
C a B
Fig - 56

In ∆OPB, we have

a
tan θ = tan ∠A = ...(1)
2x

b C
Similarly, tan ∠B = and tan ∠C = ...(2)
2y 2z

Now, the angles ∠A, ∠B and ∠C are those of a triangle so that

∠A + ∠B + ∠C = π
⇒ tan(∠A + ∠B ) = tan(π − ∠C )

= − tan ∠C

tan ∠A + tan ∠B
⇒ = − tan ∠C
1 − tan ∠A tan ∠B

⇒ tan ∠A + tan ∠B + tan ∠C = tan ∠A tan ∠B tan ∠C


Using (1) and (2) in the relation above, we obtain the required relation.

Mathematics / Circles
LOCUS 71

Example – 11

Let A( a, 0) be a point on the circle x 2 + y 2 = a 2 . Through another point B (0, b), chords are drawn to meet the
circle at points Q and R. Find the locus of the centroid of ∆AQR.
Solution : The following diagram shows an example of the situation described :

B(0, b)
Q

The points A and B are fixed.


The line BQR is of variable
slope. We need to determine
A(a, 0) the locus of the centroid
of ∆ AQR
R

Fig - 57

The equation of BQR can be written as y = mx + b where m is variable. The intersection points of
BQR with the circle (Q and R) are given by simultaneously solving the system of equations
x2 + y2 = a2
y = mx + b
Thus,
x 2 + (mx + b)2 = a 2
⇒ (1 + m 2 ) x 2 + 2mbx + b 2 − a 2 = 0 ...(1)
If we assume the co-ordinates of Q and R to ( x1 , y1 ) and ( x2 y2 ), we have from (1)
2mb
x1 + x2 = −
1 + m2
Thus,
2m 2b 2b
y1 + y2 = m( x1 + x2 ) + 2b = − + 2b =
1+ m 2
1 + m2
Let the centroid of ∆AQR be ( h, k ). We have
a + x1 + x2 0 + y1 + y2
= h, =k
3 3
⇒ x1 + x2 = 3h − a, y1 + y2 = 3k
2mb 2b
⇒ − = 3h − a, = 3k
1+ m 2
1 + m2
These two relations can be used to eliminate m and obtain a relation in ( h, k ). To specify the locus
conventionally, we use ( x, y ) instead of ( h, k ). The equation obtained is (verify) :
2 2 a2
x 2 + y 2 − ax − by + =0
3 3 9

Mathematics / Circles
LOCUS 72

Example – 12

Find the equation of the family of circles touching the lines given by

x2 − y 2 + 2 y − 1 = 0
Solution: The equation to the pair of lines can be factorised to yield the individual lines as

x − y + 1 = 0 and x + y − 1 = 0.
From the following diagram, observe carefully that for a circle to touch both the lines above, its centre
must be on one of their angle bisectors because only then will the distance of the centre from the two
lines be equal.

x + y -1 = 0
(0,λ) For a circle to
be able to touch
both the lines,
its centre must lie
on one of their
(0,1) (λ,1) angle bisectors

x - y +1 = 0

Fig - 58

The angle bisectors can be seen from inspection to be x = 0 and y = 1. Thus, the centre of the
variable circle can be assumed to be:

λ −1
(i) (0, λ ) : The distance of (0, λ ) from the two lines is
λ ∈! 2

Thus, the equation of the variable circle in this case is

(λ − 1)2
x 2 + ( y − λ )2 = ...(1)
2
|λ |
(ii) (λ ,1) : The distance of (λ ,1) from the two lines in
λ ∈! 2

Mathematics / Circles
LOCUS 73

Thus, the equation of the variable circle in this case is

λ2
( x − λ ) 2 + ( y − 1)2 = ...(2)
2

The equations (1) and (2) represent the required family of circles.

Example – 13
A circle of radius r passes through the origin O and cuts the axes at A and B. Show that the locus of the foot of the
perpendicular from O to AB is

 1 1 
( x 2 + y 2 ) 2  2 + 2  = 4r 2
x y 

Solution: Let the co-ordinates of A and B be ( a, 0) and (0, b) respectively, so that (like in Example - 8), the
equation to the variable circle becomes

x 2 + y 2 − ax − by = 0

B(0,b)
The equation for S is
x2 + y2 - ax - by = 0
P(h,k) Note that since AOB = π ,
AB is a diameter of 2
s the circle.

O A(a,0) Fig - 59

We have,

a 2 + b 2 = 4r 2 ...(1)

Let the foot of perpendicular P have the co-ordinates ( h, k ). Since OP ⊥ AB, we obtain

k b
× = −1
h −a

k h h2 + k 2 h2 + k 2
⇒ = = =
a b a 2 + b2 2r

2rk 2rh
⇒ a= ,b= ...(2)
h +k
2 2
h2 + k 2

Using (2) in (1) and ( x, y ) instead of ( h, k ), we obtain the required locus.

Mathematics / Circles
LOCUS 74

Example – 14
Find the condition so that the chord x cos α + y sin α = p subtends a right angle at the centre of the circle
x2 + y2 = a2 .
Solution:

2 2 2
x +y =a

O x

B x cos α + y sin α = p
A

Fig - 60

We can obtain the joint equation J to the pair of lines OA and OB by homogenizing the equation of the
circle using the equation of the chord AB.
2
 x cos α + y sin α 
J :x + y =a 
2 2 2
 ... (1)
 p 
For J to represent two perpendicular lines, we must have in (1),

Coeff. of x 2 + Coeff. of y 2 = 0
which upon simplification yields the required condition as

a2 = 2 p2

Example – 15

Suppose that the lines a1 x + b1 y + c1 = 0 and a2 x + b2 y + c2 = 0 intersect the co-ordinate axes at points A, B and
C , D respectively. Find the condition that must be satisfied if these four points are to be concyclic.

Mathematics / Circles
LOCUS 75

 c1   c1   c2 
Solution: The co-ordinate of A, B and C , D can be evaluated to be  − , 0  ,  0, −  ,  − , 0  and
 a1   b1   a2 
 c2 
 0, −  .
 b2 
y

B
The four points A, B and C, D
are given to be concyclic

D
x
O A C
L2: a 2 x + b2 y + c2 = 0

L1 : a1 x + b1 y + c1 = 0
Fig - 61
Instead of resorting to a detailed calculation, we simply use the result on tangents and secants that
we’ve already derived earlier:

OA ⋅ OC = OD ⋅ OB = l 2
where l is the length of the tangent drawn from O to the circle. This gives

 c1   c2   c2   c1 
 − ⋅ −  =  − ⋅ − 
 a1   a2   b2   b1 

⇒ a1a2 − b1b2 = 0

This is the required condition !

Mathematics / Circles
LOCUS 76

ASSIGNMENT

[ LEVEL - I ]
1. Find the equation of the circle passing through (–2, 1) and tangent to 3 x − 2 y − 6 = 0 at (4, 3).
2. If two circles cut a third circle orthogonally, prove that their radical axis will pass through the centre of the
third circle.
3. Find the point from which the tangents drawn to the following three circles are of equal lengths :
S1 : x 2 + y 2 − 16 x + 16 = 0
S 2 : 3 x 2 + 3 y 2 − 36 x + 8 = 0
S3 : x 2 + y 2 − 16 x − 12 y + 8 = 0
4. Prove that the length of the common chord of the two circles ( x − a )2 + ( y − b)2 = c 2 and

( x − b)2 + ( y − a)2 = c 2 is 4c 2 − 2(a − b) 2


5. Prove that the locus of the centre of the variable circle which touches externally the
x 2 + y 2 − 6 x − 6 y + 14 = 0 and the y-axis is y 2 − 10 x − 6 y + 14 = 0.
6. Find the equation of the circle whose radius is 3 and which touches internally the circle
x 2 + y 2 − 4 x − 6 y − 12 = 0 at (–1, –1).
7. Prove that the circle x 2 + y 2 + 2ax + c = 0 and x 2 + y 2 + 2by + c = 0 will touch one another if
1 1 1
2
+ 2 = .
a b c
8. If the circles x 2 + y 2 − 10 x + 16 = 0 and x 2 + y 2 = r 2 intersect in real and distinct points, prove that
2 < r < 8.
9. Find the point at which x 2 + y 2 − 4 x − 2 y − 4 = 0 and x 2 + y 2 − 12 x − 8 y − 12 = 0 touch each other.
10. Find the locus of the mid-point of a variable chord of the circle x 2 + y 2 − 2 x − 2 y − 2 which subtends an

angle at the centre.
3
11. Find the locus of the mid point of the variable chord of the circle x 2 + y 2 = a 2 which subtends a right angle
at (c, 0)
12. If the squares of the lengths of the tangents from a point P to the circles x 2 + y 2 = a 2 , x 2 + y 2 = b 2 and
x 2 + y 2 = c 2 are in A.P., show that a2, b2 and c2 are in A.P.
13. P is a variable point on y = 4. Tangents from P touch x 2 + y 2 = 4 at A and B. The parallelogram PAQB is
completed. What is the locus of Q?
π
14. The angle between T1 and T2 is , where T1 and T2 are tangents to x 2 + y 2 = 4 and x 2 + y 2 = 9
3
respectively. Find the locus of the intersection of T1 and T2 .
15. Show that tangents from (–1, 7) to x 2 + y 2 = 25 are at right angles to each other.

Mathematics / Circles
LOCUS 77

[ LEVEL - II ]

16. Find the circumcircle of the triangle formed by the three lines
L1 : x + y − 6 = 0
L2 : 2 x + y − 4 = 0
L3 : x + 2 y − 5 = 0
17. The following two circles intersect in A and B:
S1 : x 2 + y 2 + 2ax − c 2 = 0
S 2 : x 2 + y 2 + 2bx − c 2 = 0
A line through A meets one circle at P while a parallel line through B meets the other circle at Q. Show that
the locus of the mid-point of PQ is a circle.
18. A variable circle passes through the fixed point A (a, b) and touches the y-axis. Show that the locus of the
other end of the diameter through A is
( y − b) 2 = 4ax
19. Suppose that the point P(h, 0) divides a chord AB of the circle x 2 + y 2 = a 2 with slope tan θ, in the ratio
m :1. Prove that
2
 2h cos θ  h2 − a2
  =
 1+ m  m
20. Find the equation of the circle, the equation of two of whose normal is x 2 + 3 x + 6 y + 2 xy = 0 and which
is large enough to just contain the circle x 2 + y 2 = 4 x + 3 y.
21. Find the possible values of α for which the point (α − 1, α + 1) lies in the larger segment of the circle
x 2 + y 2 − x − y − 6 = 0 made by the chord x + y = 2
22. Prove that the square of the tangent that can be drawn from any point on one circle to another circle is equal
to twice the product of the perpendicular distance of the point from the radical axis of the two circles and
the distance between their centres.
23. Find the radius of the smallest circle circumscribing the following three circles :
S1 : x 2 + y 2 − 4 y − 5 = 0
S 2 : x 2 + y 2 + 12 x + 4 y + 31 = 0
S3 : x 2 + y 2 + 6 x + 12 y + 36 = 0
24. A variable chord through a fixed point Q (t, 0 )meets a fixed circle S : x 2 + y 2 = a 2 , at A and B. Find the
locus of a point P on this chord such that QA, QP and QB are in (a) A. P. (b) G.P.
25. If H is the orthocentre of a triangle ABC, prove that the radii of the circumcircles of the triangles ABC and
HBC are of the same length.

Mathematics / Circles
LOCUS 78

ANSWERS
TRY YOURSELF -I

1. [15,5]

 2 λ2 
2.  x + y 2
+ ( mx − y ) = 0
 c 

3 3 2 
3.  ( g + f 2 − c )
 4 

 x2 + y 2 − 8x − 2 y + 7 = 0 
4.  2 
 x + y − 3 x − 7 y + 12 = 0 
2

 2
1  22 
2
485 
5.  x +  +  y −  = 
 17   17  289 

6. ( x − 2 )2 + ( y + 1)2 = 1
 

7. 13 ( x 2 + y 2 ) − 2 y − 13 = 0 
 

8.  x 2 + y 2 − x − y = 0 

 −9  
10.  2 , 2  
 

Mathematics / Circles
LOCUS 79

TRY YOURSELF -II

2. [11x − 2 y − 46] = 0
3. [4 x + 3 y + 19 = 0; 4 x + 3 y − 31 = 0]

4. [± x ± y = a ]

 2  a 2 + b2  
 x + y − 2 ( x + y )  + (a + b ) = 0
2 2 2
5.
  a+b  

6. [3x – 2y] = 0

7. ( x − 6 )2 + ( y − 3)2 = 20 
 

8. [ x − 3 y = 0]
9.  x 2 + y 2 + 4 x − 6 y + 9 = 0 

TRY YOURSELF -III

1.  x 2 + y 2 − 6 x − 6 y + 9 = 0 

2. [ π / 4]
 16 31 
3.  − 21 , − 63 

4.  2ax + 2by = a 2 + b 2 + k 2 

TRY YOURSELF -IV

 2 x 2 + 2 y 2 − 18 x − 22 y + 69 = 0 
1.  2 
 x + y − 2 y − 15 = 0
2

2. ( x 2 + y 2 )( a 2 + b 2 ) = 2ab (bx + ay )
 

3. 3 ( x 2 + y 2 ) − 23 x = 4 y + 35 = 0
 

4.  x 2 + y 2 + 3 x + 2 y = 0 

5.  x 2 + y 2 + 7 x − 11 y + 38 = 0 

Mathematics / Circles
LOCUS 80

ASSIGNMENT
ANSWER
LEVEL - I

1. 7 ( x 2 + y 2 ) + 4 x − 82 y + 55 = 0
 

 10 2  
3.  3 , − 3  
 

6. 5 x 2 + 5 y 2 − 8 x − 14 y − 32 = 0 

 2 4  
9.  − , −  
 5 5  

10.  x 2 + y 2 − 2 x − 2 y + 1 = 0 

11.  2 ( x 2 + y 2 ) − 2cx + c 2 − a 2 = 0 
 

13.  2 y 2 = ( x + y ) ( x 2 + y 2 )
 

3 ( x + y ) = 76 
14.  2 2

LEVEL II

16.  x 2 + y 2 − 17 x − 19 y + 50 = 0 

17.  x 2 + y 2 + ( a + b ) x = 0 

20.  x 2 + y 2 + 6 x − 3 y − 45 = 0 

21. ( −1,1)

23. [approx 7.3]

( a ) x 2 + y 2 = tx 
24.  
(b ) x + y − 2tx + a = 0 
2 2 2

Mathematics / Circles
LOCUS 81

Mathematics / Circles

Anda mungkin juga menyukai